Paediatrics Flashcards

1
Q

A 14-year-old girl comes to see you as she has not had her periods yet. You note that her breasts are stage II and her nipples are set lateral to the mid-clavicular line. She has no pubic hair. Her weight is on the 50th centile but height is on the 9th centile. Her parents are both of average height. What is the most likely diagnosis?

a) Turner’s syndrome
b) Polycystic ovarian syndrome
c) Anorexia
d) Constitutional delay
e) Underlying undiagnosed chronic illness

A

A

Turner’s syndrome (45, XO) often goes undiagnosed in women. It is associated with neonatal hand and foot lymphoedema, webbing of the neck, short stature, wide-spaced nipples, congenital heart defects (co-arctation of the aorta), delayed or absent puberty, and infertility.

PCOS is unlikely as she hasn’t started puberty.

Constitutional delay is more common in males and is the isolated finding of delayed skeletal growth (e.g. height).

How well did you know this?
1
Not at all
2
3
4
5
Perfectly
2
Q

A 2-year-old child is referred to you by the GP because he has not started walking. His mother says that he can stand but cries to be picked up or sits down shortly. His older sister was walking by 14 months. You note that he is talking well with short two to three word phrases. He is able to build a tower of six blocks. What is your management plan?

a) Advise mum to work harder at giving him independence and follow up in 4 months
b) Request blood tests including a creatinine kinase
c) Refer for physiotherapy
d) Reassure and discharge as his development is normal
e) Refer to orthopaedics

A

B

At 2 years old, this is delay in gross motor skills without other delays. Muscular dystrophy is suspected. This usually presents with clumsiness, and a waddling gait.

If CK is raised, a muscle biopsy can confirm the diagnosis.

Diagnosis should be confirmed before referral to PT. Referral to orthopaedics would be appropriate in developmental dysplasia of the hip, but this should be confirmed first with USS.

How well did you know this?
1
Not at all
2
3
4
5
Perfectly
3
Q

You see a boy in outpatients whose parents are concerned he is not talking yet. You do a developmental assessment and find he is walking well and able to build a tower of three blocks. He will scribble but does not copy your circle. He is able to identify his nose, mouth, eyes, and ears, as well as point to mummy and daddy. You do not hear him say anything but his parents say he will say a few single words at home such as mummy, daddy, cup, and cat. He is a happy, alert child. Parents report him to be starting to feed himself with a spoon and they have just started potty training but he is still in nappies. What is the child’s most likely age?

a) 12 months
b) 15 months
c) 18 months
d) 2 years
e) 2.5 years

A

C

This child is scoring 18 months old in all areas.

Gross motor: steady walking

Fine motor: scribbling

Speech and language: identifies four body parts, at least 10 words

Social and behavioural: feeds himself with a spoon, starting to potty train

How well did you know this?
1
Not at all
2
3
4
5
Perfectly
4
Q

A one-and-a-half-year-old Caucasian child is referred to paediatrics for failure to thrive. On examination, he is a clean, well-dressed child who is quite quiet and withdrawn. He is pale and looks thin with wasted buttocks. His examination is otherwise unremarkable.
His growth chart shows good growth along the 50th centile until 6 months followed by weight down to the 9th, height down to the 25th and head circumference now starting to falter at 1.5 years. What is the most likely cause of this child’s growth failure?

a) Coeliac disease
b) Neglect
c) Constitutional delay
d) Normal child
e) Beta thalassaemia

A

A

Coeliac disease is most likely as growth was normal until the age of weaning, 6 months. Wasted buttocks are also a classic sign of coeliac disease, as is his anaemia and malnutrition.

Neglect would require MDT input to diagnose, and physical illness should always be ruled out.

Constitutional delay is common in males, and is the isolated finding of skeletal growth delay (e.g. height).

Dropping centiles is never normal.

Beta thalassaemia is uncommon in Caucasians, and also should present around 6 months of age. Without transfusions, he’d unlikely survive to one-and-a-half years.

How well did you know this?
1
Not at all
2
3
4
5
Perfectly
5
Q

A 12-year-old boy presents to his GP with left-sided unilateral breast development at stage III. He is very upset as he is being bullied at school. His mother is worried as her friend’s sister has just been diagnosed with breast cancer and wants to know if he could have breast cancer. What is the management?

a) Refer for a breast ultrasound
b) Test sex hormone levels
c) Test alpha foetoprotein
d) Reassure and explain that this is a normal part of puberty; it will resolve but the other breast may enlarge transiently as well
e) Do a fine needle aspirate on his left breast

A

D

Normal puberty in boys starts from 9 to 13 years. Testicular volume increases above 4mL, followed by penis enlargement, pubic hair growth, and lastly the growth spurt. Transient gynaecomastia is common, and doesn’t require treatment unless it doesn’t resolve.

How well did you know this?
1
Not at all
2
3
4
5
Perfectly
6
Q

You see a baby for the first baby check at 6 weeks. Mum reports no problems and he is feeding well. On examination you are unable to palpate the testicles on either side and do not feel any lumps in the groin area. He has a normal penis with no hypospadias and the anus is patent. He is otherwise a normal baby on examination. What is the most important diagnosis to rule out?

a) Klinefelter’s syndrome
b) Congenital adrenal hyperplasia
c) Undescended testicles
d) Virilised female infant
e) Testicular cancer

A

B

CAH is most commonly due to 21-hydroxylase deficiency. They often have ambiguous genitalia or bilateral undescended testes. They are at risk of a salt-losing adrenal crisis around 1 to 3 weeks of age, with vomiting, weight loss, and floppiness. If suspected, U and E, chromosomal analysis, and pelvic USS should be done.

Klinefelter’s syndrome (47XXY) typically presents as tall stature, delayed puberty, and mild learning difficulties; not undescended testes.

Undescended testicles are important to identify, but are only worrying if undescended by 2 years of age, as this increases risk of testicular cancer.

This could be a virilised female infant, but this would be secondary to another cause (e.g. CAH).

How well did you know this?
1
Not at all
2
3
4
5
Perfectly
7
Q

You see an 8-year-old boy in A and E who fell off his bike 3 days ago and scraped his left calf. The cuts are now angry, red, and painful. You note he is a big boy and plot his growth: his weight is on the 99th centile and his height is on the 75th centile. You note mild gynaecomastia and stretch marks on his abdomen, which are normal skin colour. His past medical history is unremarkable except for mild asthma. What is the most likely cause of his large size?

a) Cushing’s syndrome secondary to pituitary adenoma
b) Cushing’s syndrome secondary to becotide inhaler use
c) Obesity
d) His size is within the normal range and is a variant of normal
e) Liver failure

A

C

Advice concerning diet and exercise should be given. Children should eat a normal healthy diet (not restricted), and exercise regularly, to decrease weight as they grow.

Cushing’s syndrome would usually present with growth failure, but this boy is above average height, which is more in keeping with obesity.

How well did you know this?
1
Not at all
2
3
4
5
Perfectly
8
Q

A 16-year-old boy is brough to the GP by his parents. They are concerned he is the shortest boy in his class. He is otherwise well. His height and weight are on the 9th centile. His father plots the 75th centile and his mother on the 50th centile for adult height. On examination, his testicular volume is 8mL, he has some fine pubic and axillary hair. The rest of the physical examination is normal. On further questioning, you elicit from his father that he was a late bloomer and did not reach his full height until he was at university. What is the most likely cause of the boy’s short stature?

a) The 9th centile is a normal height and weight so there is nothing wrong with him
b) Growth hormone deficiency
c) Constitutional delay of growth and puberty
d) Underlying chronic illness should be sought
e) Anorexia

A

C

Constitutional delay is more common in males, runs in the family, and often presents with delayed puberty and growth spurt. It is an isolated skeletal growth delay.

This is not normal height for him given his parents’ height centiles.

This boy has started puberty, and is progressing normally, so it is unlikely to be growth hormone deficiency

How well did you know this?
1
Not at all
2
3
4
5
Perfectly
9
Q

A mother comes to see you with her 2-year-old daughter, Stacey, out of frustration that her daughter is so ill behaved. She does not know how to make her listen and is worried that she is going to get hurt. Yesterday she ran ahead and did not stop when her mother called to her. She ran into the street and was hit by a cyclist, but fortunately he was OK and Stacey had only a few cuts and scrapes and seems alright. On questioning, you hear other stories of a naughty child. She is active and eats well, feeding herself a lot now, but her mother does say she gets frequent coughs and colds. Her mother says that Stacey only says about 5-10 words and only she can understand what Stacey says. What is the best next management?

a) Ask the health visitor to visit mum for parenting advice and support
b) Order blood tests for full blood count to check for leukaemia as she has recurrent coughs and colds
c) Give Stacey a tetanus shot to cover her after her fall the day before
d) Refer for a hearing test
e) Tell Stacey that she needs to listen to her mother and not have any more accidents

A

D

Stacey has isolated speech delay. This, combined with her poor attentiveness, suggests hearing difficulty.

Commonly, this is because of glue ear, which is supported by her recurrent coughs and colds. These coughs and colds are not worrying, as they are not severe, and most children have on average 8 illnesses a year, so she doesn’t need testing for immune deficiencies.

How well did you know this?
1
Not at all
2
3
4
5
Perfectly
10
Q

An older mother books in to see you after attending the health visitor for a weight check at 2 months for her first child. She and her husband have had a hard time coming to terms with their daughter’s diagnosis of Down’s syndrome. She is relieved that the appointment with the cardiologists went well and the heart is normal, however they have a lot of trouble getting her to take the whole bottle. She was slow to regain her birth weight, and, looking at the plotted weight yesterday, she is not growing along her birth centile. The mother is worried she is not doing a good enough job She is not vomiting except for small possets after she feeds, is passing urine, and opening her bowels. The red book growth chart shows the weight to be falling off centiles. What is the most appropriate management?

a) Contact the cardiologists in light of the poor feeding and slow weight gain for a second opinion, because babies with Down’s syndrome are at high risk of heart problems and they may have missed it
b) Refer to the dietician for nutritional support
c) Replace the growth chart in their red book with a Down’s syndrome growth chart, reassure mum by re-plotting her growth and explain she is normal but arrange to review again
d) Tell the mother to try a different milk and come back in 2 weeks
e) Advise the mother to change to a faster flow teat for their bottles so that she takes her feed better

A

C

Children with Down’s syndrome have different growth patterns to other children.

Even if there was a lack of weight gain, it’s unlikely that this is to do with cardiology if they’ve given the all clear.

Dietician advice is warranted if there is still failure to thrive on the Down’s syndrome growth charts, but should be given with appropriate investigations for a cause.

Different milks are unlikely to help as there doesn’t appear to be an intolerance.

A faster teat could make things worse, as Down’s syndrome is associated with hypotonia and swallowing difficulties.

How well did you know this?
1
Not at all
2
3
4
5
Perfectly
11
Q

Which child should be moved to the resuscitation area for urgent management in A and E?

a) A miserable 2-year-old with fever and vomiting; temperature 38.5C, HR 150bpm, RR 42/m, CRT 2 - 3s; alert and clinging onto his father; just been given paracetamol and started on a fluid challenge with oral rehydration salts 5m ago by the triage nurse
b) A quiet 4-year-old brought in with an asthma attack, sitting upright with a RR 25/m, HR 162bpm, CRT 3s, subcostal recessions, and poor air entry on chest auscultation following a salbutamol nebuliser
c) An 8-year-old, known diabetic, brought in vomiting wh her glucose reader saying HI. She is able to tell you her history and has a HR 120bpm, RR 25/m, CRT <2s
d) An alert 3-year-old who has had a cough and cold for the past 3 days which is keeping him up at night, and mum noticed a rash on his neck and face which did not disappear when she pressed a glass tumbler against it. His temperature is 37.8C, HR 110bpm, RR 30/m, CRT <2s.
e) A 15-year-old, known to social services for a family history of domestic abuse, brought into A and E by her best friend after she admitted taking 20 paracetamol tablets 4 hours ago. She is alert but does not make eye contact, her HR is 98bpm, RR 20/m, CRT <2s

A

B

This child is tachypnoeic, tachycardic, and has circulatory compromise. She has not responded to initial treatment, so needs urgent management.

Child A is unwell but has only just been given initial treatment, so should be observed on his response before urgent management is required.

Child C may have DKA, but remains alert with no evidence of circulatory compromise.

Child D needs reviewing, however he is not septic, so is not an urgent case.

Child E is at risk of becoming very sick, but is currently stable.

How well did you know this?
1
Not at all
2
3
4
5
Perfectly
12
Q

A 4-year-old child has been losing weight recently and has been vomiting for the past 24 hours, unable to eat anything. His mother has brought him into A and E out of concern as he seems confused. The triage nurse has taken him to the resuscitation room and asked for your help. On examination he is drowsy, has a HR 150bpm, RR 60/m, CRT 5s. He has subcostal recessions and good air entry bilaterally with no added sounds. He moans when you examine his abdomen, but there are no masses. You put in a cannula and take bloods. The venous blood gas shows:

pH 7.12
PCO2 2.3kPa
PO2 6.7kPa
HCO3 15.3 mmol/L
BE -8.6
Glucose 32.4mmol/L

What is the most likely diagnosis and what is the first management step?

a) Diabetic ketoacidosis, start an insulin infusion
b) Diabetic ketoacidosis, give a fluid bolus
c) Pneumonia, start IV co-amoxiclav
d) Ruptured appendix, give a fluid bolus and book the emergency operating theatre
e) Gastroenteritis with severe dehydration, give a fluid bolus

A

B

This child is tachycardic, tachypnoeic, and has circulatory compromise. His bloods show a metabolic acidosis and hyperglycaemia. This is DKA.

Treatment of DKA involves correction of dehydration and glycaemic index over 2 days to prevent risk of brainstem demyelination due to rapid shifts in CSF salts. Therefore, a fluid bolus is the first step, followed by rehydration, gradual insulin and potassium.

How well did you know this?
1
Not at all
2
3
4
5
Perfectly
13
Q

An 8-year-old known asthmatic is brought into A and E by ambulance as a ‘blue call’. He has been unwell with an upper respiratory tract infection for the past 2 days. For the past 24h, his parents have given him 10 puffs of salbutamol every 4 hours, his last dose being 90m ago. The ambulance staff have given him a nebuliser, but he remains agitated with a HR 155bpm, RR 44/m, and sub- and intercostal recessions. On auscultation, there is little air movement heard bilaterally. Saturations in air are 85%. He is started on ‘back to back’ nebulisers with high flow oxygen. How severe is his asthma exacerbation, and what other bedside test would support this?

a) Moderate, venous blood pH 4.4, gas PCO2 3.1kPa
b) Severe, peak flow <33% expected
c) Severe, venous blood pH 4.4, gas PCO2 3.1kPa
d) Life-threatening, peak flow <33% expected
e) Life-threatening, venous blood pH 4.4, gas PCO2 3.1kPa

A

D

Asthma is assessed by best or expected PEF. Though blood gas is useful, it is not used to assess exacerbation.

Life-threatening: SpO2 <92%; PEF <33%; silent chest; cyanosis; poor respiratory effort; hypotension; exhaustion; confusion

Severe: SpO2 <92%; PEF 33–50%; HR >125 (>5 years) or >140 (1-5 years); RR >30 breaths/min (>5 years, or >40 (1–5 years); can’t complete sentences in one breath or too breathless to talk or feed

Moderate asthma gives a PEF of 50-75%.

How well did you know this?
1
Not at all
2
3
4
5
Perfectly
14
Q

The A and E triage nurse asks you to look at a 3-year-old child with a short history of waking up this morning unwell with a cough and fever. She looks unwell, HR 165bpm, RR 56/m, sats 96%, temperature 39.3C, CRT 4s. She has a mild headache but no photophobia or neck stiffness and you notice a faint macular rash on her torso and wonder if one spot is non-blanching. You ask the triage nurse to move her to the resuscitation area and call your senior to review her. Fifteen minutes later your senior arrives and the spot you saw on the abdomen is now non-blanching and there is another spot on her knee. What are the three most important things to give her immediately?

a) High flow oxygen, IV fluid bolus, IV ceftriaxone
b) IV fluid bolus, IV ceftriaxone, IV methylprednisolone
c) High flow oxygen, IV ceftriaxone, IV fresh frozen plasma
d) IV fluid bolus, IV ceftriaxone, IV fresh frozen plasma
e) High flow oxygen, IV ceftriaxone, IV methylprednisolone

A

A

This is meningococcal sepsis.

Sick children should always be given oxygen. She shows signs of hypovolaemia, so a fluid bolus is necessary. IV ceftriaxone should be given as soon as a blood culture is taken.

Steroids have been shown to help in cases of meningitis with or before the first dose of antibiotics. In this case, there is no sign of meningitis, as she has no photophobia or neck stiffness, so steroids are not necessary.

Fresh frozen plasma is given if clotting results show disseminated intravascular coagulation, to replace clotting factors.

How well did you know this?
1
Not at all
2
3
4
5
Perfectly
15
Q

A 9-year-old boy is brought in by ambulance having been hit by a car while playing football in the street. You have been assigned to do the primary survey in resus when the ambulance arrives The patient is receiving oxygen, crying for his mummy and holding his right arm, but able to move over from the stretcher to the bed when asked. Which is the correct examination procedure?

a) The trachea is deviated to the right. On auscultation you hear decreased air entry on the left. Percussion note is hyper-resonant on the left. He is tachycardic and his heart sounds are muffled, heard loudest at the right lower sternal edge. You ask for a left-sided thoracocentesis.
b) You introduce yourself and tell him that you will be gentle but need to check that he is okay. You see his left wrist is deformed and swollen, and check the fingers, which are cool, and note the CRT is 4s. He is able to feel you touching him and moans when you examine the wrist. You call for an x-ray to assess the probable fracture in the wrist.
c) You introduce yourself and tell him that you will be gentle but need to check if he is okay. You listen for equal air entry and think there is decreased air entry on the left, but there is air entry on the right. He is tachypnoeic and has a pulse which is tachycardic. His CRT is 4s. You expose his abdomen and notice bruising and grazes to the left side. He moans as you palpate the left upper quadrant and has guarding. You ask for an IV cannula or IO needle and a 20ml/kg fluid bolus while organising an urgent CT chest and abdomen.
d) You introduce yourself and tell him that you will be gentle but need to check if he is okay. He is tachypnoeic. The trachea is deviated to the right. On auscultation you hear decreased air entry on the left. Percussion note is hyper-resonant on the left. He is tachycardic and his heart sounds are muffled, heard loudest at the right lower sternal edge. You ask for a left-sided thoracocentesis.
e) You listen for equal air entry and think there is decreased air entry on the left but there is air entry on the right. He is tachypnoeic and has a pulse which is tachycardic. His CRT is 4s. You expose his abdomen and notice bruising and grazes to the left side. he moans as you palpate in the left upper quadrant and has guarding. You ask for an IV cannula or IO needle and a 20mL/kg fluid bolus while organising an urgent CT chest and abdomen.

A

D

You should introduce yourself, and proceed in an ABC approach, with management of each section before repeating the full ABC examination, as in the fourth scenario.

The first scenario is incorrect because of lack of introduction.

The second skips ABC and goes straight to D.

The third ignores the problem identified in B (pneumothorax) and skips to correcting C.

The last is incorrect because you fail to introduce yourself, and fail to address the pneumothorax.

How well did you know this?
1
Not at all
2
3
4
5
Perfectly
16
Q

A 6-year-old boy with a history of anaphylaxis to peanuts is brought in by ambulance unconscious. He was attending a children’s birthday party. His mother says there was a bowl full of candy and he may have eaten a Snickers bar but she is not sure and she did not have his EpiPen with her. His face and lips are swollen and erythematous. He is still breathing, but weakly, and there is wheeze. His pulse is tachycardic and thready. Which type of shock is this?

a) Hypovolaemic
b) Distributive
c) Septic
d) Cardiac
e) Obstructive

A

B

Shock is inadequate perfusion of tissues insufficient to meet cellular metabolic needs. This child has anaphylaxis and has not had IM adrenaline to prevent capillary leakage into the airway. Oedema causes airway closure, and leakage causes intravascular hypovolaemia, or distributive shock.

Hypovolaemic shock is due to haemorrhage or dehydration.

Septic shock requires fever and infection, causing widespread vasodilation.

Cardiac shock is a result of cardiac insufficiency to meet the demands of the body. This is rare in paediatrics, but could occur due to congenital heart disease or Kawasaki’s disease.

Obstructive shock is due to blockage of blood flow from the heart due to cardiac tamponade, or tension pneumothorax.

How well did you know this?
1
Not at all
2
3
4
5
Perfectly
17
Q

A 13-month-old is brought in having had a blue floppy episode at home lasting 1 minute. While you are taking a history from the mother, you notice the baby has gone blue again and seems to be unconscious in her arms. You call for help and place the baby on the examination table. There is no obvious work of breathing. The nurses bring the crash trolley and give you a bag valve mask, which they are connecting to the oxygen. You give two inflation breaths but do not see the chest rise. You reposition the airway and this time the breaths go in. You feel for a pulse and there is none. When asked to do CPR, the nurse asks for direction on how many breaths and compressions you both need to do.

a) Two inflation breaths per 30 compressions
b) Two inflation breaths per 15 compressions
c) Continuous inflation breaths about 10-12 per minute and compressions 100-120 per minute
d) One inflation breath per five compressions
e) Two inflation breaths per five compressions

A

B

How well did you know this?
1
Not at all
2
3
4
5
Perfectly
18
Q

A 10-year-old child is brought in by ambulance with seizure activity. His mother reports it starting 30 minutes ago in his right arm and quickly became generalised tonic clonic jerking. She have him his buccal midazolam after the first 5 minutes and called an ambulance when he did not respond after another 5 minutes. The ambulance crew gave him rectal diazepam on arrival at 15 minutes into the seizure. He is receiving high flow oxygen via face mask and continues to convulse. The mother tells you that he was weaned from his long-term seizure medication, phenytoin, 2 weeks ago and that he has had a cold for the past 2 days. What is the next step in management?

a) Gain IV or IO access and administer lorazepam
b) Gain IV or IO access and administer ceftriaxone
c) Repeat the rectal diazepam
d) Gain IV or IO access and start a phenytoin infusion
e) Gain IV or IO access and start a phenobarbital infusion

A

D

This is status epilepticus, a continuous seizure lasting over 30m, due to either epilepsy or fevers.

If a seizure lasts more than 5 m, buccal or rectal diazepam should be administered, or IV/IO lorazepam if necessary. If seizure continues for another 10m, a second dose of benzodiazepines should be given, but no more than 2 can be given due to risk of respiratory depression. The next step is IV/IO phenytoin infusion over 20m, unless the child is already on phenytoin, in which case phenobarbital should be given. If still no response, intubation, ventilation, and rapid sequence induction is indicated.

How well did you know this?
1
Not at all
2
3
4
5
Perfectly
19
Q

A 3-year-old boy is brought in by ambulance fitting. You are assigned to get the history from the father. Harry is normally fit and well with no significant past medical history or allergies. He is up to date with his immunisations and has been growing and developing normally. His behaviour has been difficult for the past 2 weeks since the birth of his little sister. Mum has been unwell as she developed HELLP syndrome and was in hospital for a week following the delivery. Yesterday, he was quite unwell with a tummy bug, vomiting and had black diarrhoea. That evening they found a mess he had made in the bathroom with all of his mum’s things strewn over the floor including her tablets from the hospital. By that time, Harry was getting better so they did not think anything of it. Today he has been acting strangely and has been difficult to understand. He then became lethargic at about 4pm and started fitting 15m ago. What is the most likely diagnosis?

a) Paracetamol overdose
b) Aspirin overdose
c) Tricyclic antidepressant overdose
d) Bleach intoxication
e) Iron overdose

A

E

Iron overdose presents classically in 2 phases: early vomiting and diarrhoea due to gastric irritation (possible haematemesis or malaena), followed by 24h improvement, then deterioration with liver failure, drowsiness, and coma. Women with HELLP often have significant post-partum haemorrhage, and so require iron supplements.

How well did you know this?
1
Not at all
2
3
4
5
Perfectly
20
Q

A 6-year-old boy with a history of asthma and eczema is brought in to A and E from a local restaurant. He is on high flow facial oxygen with significant facial oedema and generalised erythema. On auscultation there is widespread wheeze for which the ambulance crew gave a salbutamol nebuliser. What is the next step in management?

a) Insert an IV line and give 10mg slow IV antihistamine
b) Insert an IV line and give 100mg slow IV hydrocortisone
c) Insert an IV line and give 200ug of 1:10,000 IV adrenaline
d) Give IM 1:1000 adrenaline, 250ug
e) Repeat the salbutamol nebuliser and call for an anaesthetist for intubation

A

D

As well as giving IM adrenaline, an emergency call to the anaesthetist should be made, and antihistamines and steroids should be given to prevent capillary leakage and reduce inflammation. Steroids will also help prevent late type IV hypersensitivity, so children should be observed and sent home with 2 further doses of prednisolone.

How well did you know this?
1
Not at all
2
3
4
5
Perfectly
21
Q

A newborn baby is born to non-consanguineous parents. She is noted to have puffy feet on her 1st day check. She weighs 2.0kg with widely spaced nipples and absent femoral pulses. You have asked your registrar to review her as you think she may have Turner’s syndrome. She agrees and asks you to send blood tests for karyotyping. Which is the chromosomal diagnosis of Turner’s syndrome?

A) 47XXY
B) 45YO
C) 46XY
D) 46XX
E) 45XO
A

E

47XXY is Klinefelter’s syndrome, presenting in boys with tall stature, delayed puberty, and gynaecomastia.

45YO is not compatible with life.

46XY and 46XX are normal karyotypes for boys and girls respectively.

How well did you know this?
1
Not at all
2
3
4
5
Perfectly
22
Q

A 15-year-old boy was diagnosed with Down’s syndrome at birth. He is short for his age, had cardiac surgery as a baby, has treatment for hypothyroidism, and now attends mainstream school with some support. His parents are enquiring now about what complications he faces. Which of these is not a recognised complication of Down’s syndrome?

A) Retinoblastoma
B) Atrioventricular septal defect (AVSD)
C) Type 1 diabetes
D) Leukaemia
E) Alzheimer's disease
A

A

Trisomy 21 has effects in many systems:

Cardiac (VSD, AVSD, Tetralogy of Fallot)
Endocrine (hypothyroidism, Addison’s disease, T1D)
Ocular (cataracts)
Malignancy (leukaemia)
GI (duodenal atresia, Hirschprung’s disease)
Musculoskeletal (atlanto-axial instability)
Neurological (Alzheimer’s, hypotonia, developmental delay, mean IQ=50)

How well did you know this?
1
Not at all
2
3
4
5
Perfectly
23
Q

A baby is born and you are asked to do the baby check at 6 hours post-natal age. You go to see the baby and mum states that he has not yet had a feed. You advise they stay in hospital until the feeding is established. This is the first child of non-consanguineous parents. On day 4, when you review the baby, he has still not had an adequate intake, has lost over 10% in birth weight and is markedly hypotonic. Your consultant asks you to request genetic testing for Prader-Willi syndrome. What is the inheritance of Prader-Willi syndrome?

A) X-linked
B) Imprinting
C) Monosomy
D) Microdeletion
E) Trisomy
A

B

How well did you know this?
1
Not at all
2
3
4
5
Perfectly
24
Q

A 5-day-old baby who is formula fed is on the neonatal unit being treated for sepsis secondary to an E. coli urinary tract infection. He has been on antibiotics for 5 days. He is still unwell and vomiting. The parents are consanguineous, and this is their first child. He has repeat blood and urine cultures taken. Urine reducing substances are positive. What is the most likely underlying diagnosis?

A) Fructose intolerance
B) Galactosaemia
C) Phenylketonuria
D) Lactose intolerance
E) Glycogen storage disease
A

B

Galactosaemia is due to a deficiency in galactose-1-phosphate uridyl transferase. It causes vomiting, cataracts, and recurrent E. coli sepsis.

Fructose intolerance can cause vomiting and metabolic acidosis, but without the E. coli sepsis.

PKU results in developmental delay and musty smelling urine.

Lactose intolerance presents with diarrhoea and poor weight gain, but is unlikely due to high levels of lactase at birth.

GSDs present with liver, muscle, and cardiac defects.

Consanguinuity is defined as between a couple that are second cousins or closer.

How well did you know this?
1
Not at all
2
3
4
5
Perfectly
25
Q

A 10-year-old boy is brought to the GP with tall stature. He is taller than his peers at school. His arm span is greater than his height, he has long, thin fingers, scoliosis, and pectus excavatum. He is also concerned that he gets short of breath at school during PE lessons. You refer him for an echocardiogram and chest x-ray. You make a clinical diagnosis of Marfan’s syndrome. What is the inheritance of Marfan’s syndrome?

A) X-linked recessive
B) Autosomal recessive
C) Sporadic
D) X-linked dominant
E) Autosomal dominant
A

E

How well did you know this?
1
Not at all
2
3
4
5
Perfectly
26
Q

A pregnant woman seeks advice from you regarding her condition and its impact on the pregnancy and risk to the baby. She has phenylketonuria (PKU) and has been on a phenylalanine-free diet for life. She was told that it was very important during her pregnancy to be compliant with this diet. She would like to know how the baby will be tested for the condition as she is aware that is an inherited condition. What is the initial investigation you will advise?

A) Serum tyrosine levels
B) Genetic screening
C) Serum phenylalanine levels
D) Urine phenylketones
E) Newborn blood spot screening
A

E

How well did you know this?
1
Not at all
2
3
4
5
Perfectly
27
Q

You see an 18-year-old boy who is the first child of his African parents and was born in Kenya before moving to the UK 1 year ago. He has white skin and pink irises. He was diagnosed with oculocutaneous albinism at birth. He has difficulty with his sight, but has recently developed a skin lesion on his face. His mother has brought him to his GP as it has recently started to increase in size. On examination, you note, is an elevated, 3 cm diameter lump on the left of his nose. It has irregular edges, is firm and immobile, and pigmented in areas. What is the likely diagnosis?

A) Benign naevus
B) Scar from a healing wound
C) Malignant tumour
D) Abscess
E) Wart
A

C

A benign naevus, or mole, would be static in size, smooth and regular edged.

There is no evidence of trauma or infection, so scars and abscesses would be unlikely.

A wart cause by HPV6 or 11 is possible, but rarely on the face and less likely given the medical background.

How well did you know this?
1
Not at all
2
3
4
5
Perfectly
28
Q

You are asked to see a 3-day-old baby on the post-natal ward. The baby was born at term and is the first child of consanguineous parents. The baby is drowsy and vomiting, with no fever, rash, or diarrhoea. On examination, the baby is noted to have ambiguous genitalia. You do some blood tests: WCC 5x10^9/L; CRP 2mg/L; Na 125 mmol/L; K 8 mol/L; glucose 1.7mmol/L. 17-OH level progesterone is low. You make a diagnosis of congenital adrenal hyperplasia. What is the best initial management plan?

A) IV hydrocortisone
B) IV dextrose
C) IV dextrose and IV hydrocortisone
D) IV 0.9% saline
E) IV 3% saline and IV hydrocortisone
A

C

CAH is a lack of the enzymes that metabolise testosterones to cortisol. Therefore, there is a build-up in testosterones (leading to masculinisation) and a lack of cortisol (salt-loss, low glucose, vomiting).

Glucose and steroid replacement is required. Sodium should not be replaced too quickly due to risk of central pontine myelinolysis.

How well did you know this?
1
Not at all
2
3
4
5
Perfectly
29
Q

A 10-year-old boy is brought to the paediatric outpatient department for a review of his height. He was found to be on the 0.4th centile, and his mid-parental height is the 98th centile. He also has widely spaced nipples, wide carrying angle, hypogonadism, pulmonary stenosis, and developmental delay. What is the most likely diagnosis?

A) Angelman's syndrome
B) William's syndrome
C) Turner's syndrome
D) Prader-Willi syndrome
E) Noonan's syndrome
A

E

Noonan’s syndrome presents similarly to Turner’s syndrome, but is of autosomal dominant inheritance, so can present in boys.

Angelman’s syndrome is loss of maternal 15q, causing developmental delay and happy demeanour.

William’s syndrome is an autosomal dominant condition with typical facial features, aortic stenosis, and developmental delay.

Prader-Willi syndrome is loss of paternal 15q, causing poor feeding and weight gain in the neonatal period.

How well did you know this?
1
Not at all
2
3
4
5
Perfectly
30
Q

A 2-week-old baby was referred to the prolonged jaundice clinic by the community midwife. The pregnancy was unremarkable, and she was born at term with no antenatal abnormalities on USS or blood serology. She is now 17 days old and has been jaundiced since day 5 of life, and never required phototherapy. She is breastfed and feeds 3-hourly for 20-25 minutes. She is afebrile and not lethargic. Her mother reports that the stools are pale and she has dark coloured urine. The bilirubin is 300 umol/L, and conjugated bilirubin 100 umol/L. What is the most important diagnosis to exclude?

A) Breast milk jaundice
B) ABO incompatibility
C) Biliary atresia
D) Neonatal hepatitis
E) Hypothyroidism
A

C

Jaundice is physiological in between 24h and 2 weeks.

If it is early, it is due to haemolysis (ABO incompatibility, rhesus disease, G6PD, spherocytosis). If it persists, unconjugated jaundice can be a result of infection, haemolysis, hypothyroidism, or galactosaemia. Conjugated jaundice, as here, is caused by biliary atresia or neonatal hepatitis.

How well did you know this?
1
Not at all
2
3
4
5
Perfectly
31
Q

A 1-day-old baby is on the post-natal ward. You are asked to review her as she is febrile and lethargic. On examination, she is tachycardic, has a capillary refill time of 3 seconds centrally, and reduced urine output. Her blood culture 24h later grows Gram-positive cocci. What is the most likely causative organism?

A) Streptococcus pneumoniae
B) Staphylococcus aureus
C) Group B Streptococcus
D) Streptococcus biridans
E) Group A Streptococcus
A

C

The most common causes of neonatal sepsis are GBS (Streptococcus agalactiae), E. coli, and Listeria monocytogenes.

How well did you know this?
1
Not at all
2
3
4
5
Perfectly
32
Q

A preterm baby is born at 25 + 6 weeks gestation. he is delivered by Caesarean section due to maternal pre-eclampsia. He is intubated at birth and given surfactant via the endotracheal tube. He is ventilated and commenced on IV dextrose. After 4 hours of age, he has increased work of breathing, with intercostal and subcostal recession, and a respiratory rate of 60/min. A CXR shows a ground glass pattern in both lung fields. He has no audible murmur. He is afebrile. You diagnose respiratory distress syndrome. What is the aetiological factor responsible for respiratory distress syndrome?

A) Pneumonitis
B) Lung hypoplasia
C) Surfactant deficiency
D) Immature lung parenchyma
E) Infection with group B Streptococcus
A

C

RDS is secondary to surfactant deficiency.

Pneumonitis is a result of meconium aspiration syndrome.

Lung hypoplasia is a result of renal problems or diaphragmatic hernia, where the underlying lung is unable to develop due to compression.

The lung parenchyma is structurally immature but functional.

There are no signs of infectious causes.

How well did you know this?
1
Not at all
2
3
4
5
Perfectly
33
Q

A pre-term baby is now 25 + 7 weeks corrected gestation. He is on the neonatal unit being cared for while his mother recovers on ITU after he was born secondary to an eclamptic seizure. He has been receiving formula milk as the parents have not consented to donor breast milk. He has been having bilious aspirates form his nasogastric tube, and today his abdomen is very distended and tense. He has had one episode of bloody stools. You are going to treat him for necrotising enterocolitis (NEC). What is the best initial management plan?

A) Conservative management, observe and reassess
B) Nil by mouth (NBM), IV antibiotics, and emergency exploratory laparotomy
C) IV fluids, emergency laparotomy, and bowel resection
D) IV fluids and IV antibiotics
E) NBM, IV fluids, abdominal X-ray, and surgical review

A

E

NEC is high risk in this situation due to the formula feeding, prematurity, and potential ischaemia at birth. Other risk factors include IUGR, polycythaemia, and PDA. Active management is necessary, including immediate review by the surgeon. An AXR showing intramural gas is pathognomic of NEC, and would lead to laparatomy and bowel resection. Initial management, however, is surgical review.

How well did you know this?
1
Not at all
2
3
4
5
Perfectly
34
Q

A 3-day-old baby is seen by the midwife for a routine post-natal review. She notices that he is very floppy and his mother has raised concerns about his poor feeding. He has a protruding tongue, epicanthic folds, low set ears and sandal gap toes. She explains to the parents that she thinks he may have Down’s syndrome and refers him to the paediatrician. What is the diagnostic test for Down’s syndrome?

A) Serum alpha fetoprotein, beta human chorionic gonadotrophin, oestriol, inhibin
B) Gene mutation analysis
C) Clinical diagnosis
D) Karyotype
E) FISH
A

D

How well did you know this?
1
Not at all
2
3
4
5
Perfectly
35
Q

A baby is born by emergency C-section due to foetal tachycardia. His delivery was uneventful and you are asked to see him 5 hours later on the post-natal ward. He has just taken his first feed and has been coughing and spluttering since. He had an episode with blue lips transiently and this has now improved; his oxygen saturations are 97% in air and he is pyrexial. On examination, you note other features including vertebral and limb abnormalities, imperforate anus, pan systolic murmur at the lower left sternal edge, and renal anomalies noted on antenatal scans. What is the most likely cause for his coughing episode?

A) Cleft palate
B) Tracheoesophageal fistula
C) Choanal atresia
D) Incoordinated swallowing reflex
E) Pneumonia
A

B

This is a VACTERL association (Vertebral anomalies, Anal imperforation, Cardiac problems, Tracheo-oEsophageal fistula, Renal problems, Limb defects).

Cleft palate gives a similar history but would be noticed on examination.

Choanal atresia is associated with CHARGE (Coloboma, Heart defects, Atresia choanae, Retardation of growth and development, Genitourinary abnormalities, Ear anomalies).

Incoordinated swallowing occurs in neurological or neuromuscular problems, but there is no evidence of these.

Pneumonia can occur due to a tracheooesophageal fistula, but there’s no evidence so far of this.

How well did you know this?
1
Not at all
2
3
4
5
Perfectly
36
Q

A term baby is awaiting his discharge check when you are called to see him at 10 hours of age. His mother reports that he has turned a dusky colour and is not as alert as he has been. On examination, he has central cyanosis, pulse 150 bpm regular, and both brachial and femoral pulses are palpable. He has normal heart sounds with no murmur. His oxygen saturations are 65% in air. What is the most likely underlying diagnosis?

A) Transposition of the great vessels
B) Ventricular septal defect (VSD) 
C) Tetralogy of Fallot
D) Aortic stenosis
E) Coarctation of the aorta
A

A

This is congenital cyanotic heart disease, which can be transposition of the great arteries and Tetralogy of Fallot. The first presents as described, whereas the latter tends to present around 6 months with cyanotic spells. Transposition is only compatible with life in the presence of a mixing defect (VSD, ASD, PDA), so prostaglandins must be given to keep the duct open before surgical intervention.

VSD, aortic stenosis, and coarctation of the aorta are acyanotic conditions.

How well did you know this?
1
Not at all
2
3
4
5
Perfectly
37
Q

A 12-hour-old baby on the post-natal ward has just had a seizure lasting 2 minutes. It resolved spontaneously and was generalised in nature. Her mother had gestational diabetes and poor glucose control in pregnancy. The baby’s birth weight was 5kg. There were no abnormalities noted on antenatal USS or maternal serology. On examination, she has no dysmorphic features, and handles well. What initial blood tests would you do for the baby?

A) Liver function tests
B) Boehringer Mannheim (BM) glucose
C) Full blood count, CRP
D) Electrolytes
E) Calcium, magnesium
A

B

Infants of diabetic mothers are at risk of congenital malformations, hypoglycaemia, and macrosomia. A high level of glucose from the mother reduces the infant’s insulin levels, so leading to hypoglycaemia at birth.

Seizures can also be a result of infection or electrolyte disturbances, but this is less likely given the history.

How well did you know this?
1
Not at all
2
3
4
5
Perfectly
38
Q

A pregnant woman is admitted to the labour ward for an elective C-section at 38 weeks for her baby who had an antenatal diagnosis of gastroschisis. The paediatric team are called to attend the delivery. The baby is born in good condition with no resuscitation required. He is taken to the neonatal unit for further care. Which of these is a complication of gastroschisis?

A) Dehydration
B) Hyperthermia
C) Necrotising enterocolitis (NEC) 
D) Fluid overload
E) Hypernatraemia
A

A

Gastroschisis is a rare condition in which the bowel herniates through the anterior abdominal wall. It is not covered by a membrane (as in exomphalos), so fluid, electrolyte, and heat losses are common (not overload, hypernatraemia, and hyperthermia). NEC is not more likely in children with gastroschisis.

How well did you know this?
1
Not at all
2
3
4
5
Perfectly
39
Q

You are called to see a baby who has just been born at 39 weeks’ gestation, as the midwife thinks he is small and should be admitted to the neonatal unit for his care. You review the baby. His weight is 1.8kg, below the 0.4th centile, and his head circumference is 35cm - 50th centile. He has no dysmorphic features. Which is the most likely cause of this IUGR?

A) Chromosomal anomaly
B) Maternal smoking
C) Congenital infection
D) Maternal alcohol use
E) Placental insufficiency
A

E

This child has asymmetrical IUGR, suggesting the insult occurred late in pregnancy with head growth sparing. Causes of placental insufficiency include maternal diabetes or pre-eclampsia, with effects on placental microvasculature. The other options would more likely cause symmetrical IUGR, present from the first trimester.

How well did you know this?
1
Not at all
2
3
4
5
Perfectly
40
Q

A mother brings her 4-week-old baby to see you for the third time. He was born at term by normal vaginal delivery with no complications. You started him on anti-reflux medicine last week but it has not helped. He is now vomiting his whole feed, and is becoming lethargic and passing less urine and stool. His mother says he is even hungry after he vomits. The practice nurse has weighed him and he has lost 200g since last week. His mother was breastfeeding him while waiting to be seen, and as you got to examine him, the baby has a large milky vomit, which cascades over the clinic floor. What is the most likely diagnosis?

A) Gastroenteritis
B) Volvulus
C) Necrotising enterocolitis (NEC) 
D) Intussusception
E) Pyloric stenosis
A

E

Pyloric stenosis presents at 1 month, more commonly in males, with projectile vomiting of increasing size. It’s cause by hypertrophy of the pylorus, and requires surgical intervention.

Gastroenteritis presents with a short history of vomiting and diarrhoea, and can be caused by using unsterilised water to make formula.

Volvulus and intussusception cause colicky pain, absolute constipation, abdominal distension, and bile-stained gastric aspirates.

NEC causes distension with bile-stained aspirates, and is predominantly in premature infants.

How well did you know this?
1
Not at all
2
3
4
5
Perfectly
41
Q

A 15-month-old girl has come to see you with her father. The family are worried that she has had diarrhoea for more than a month, occasional vomiting and is losing weight. She used to be a happy, interactive baby, but now seems lethargic and miserable most of the time.. She has no significant past medical history, the rest of the family are well, and there is no history of travel. Her mother has well-controlled type 1 diabetes. The child’s weight at 6 months in the personal child health record (‘red book’) was on the 50th centile, but now is just below the 9th. What is the most likely diagnosis?

A) Crohn's disease
B) Ulcerative colitis
C) Coeliac disease
D) Irritable bowel syndrome
E) Giardiasis
A

C

Coeliac disease is an autoimmune sensitivity to dietary gluten causing villous atrophy of the small intestine. After weaning, presentation is of diarrhoea, distension, failure to thrive, and wasting.

Crohn’s disease and UC tend to present in teenagers or older.

IBS again presents much later, and is a diagnosis of exclusion.

Giardiasis would be more likely in a travel history or infectious picture.

How well did you know this?
1
Not at all
2
3
4
5
Perfectly
42
Q

A 13-year-old boy is brought to see you as he has been complaining of abdominal pain and is increasingly tired. On examination, you note some early clubbing, and erythematous palms. His conjunctivae look pale. he has one or two spider naevi on his chest. His abdomen is soft with mild tenderness in the epigastrium and right upper quadrant. The liver is palpable at 1cm and you feel the splenic tip. He has normal bowel sounds and no bruits. On slit lamp examination of his eyes, an amber ring is noted around the cornea. What is the most likely diagnosis?

A) Abdominal tuberculosis
B) Cystic fibrosis
C) Wilson's disease
D) Acute hepatitis A
E) Glandular fever
A

C

GI causes of clubbing are malignancy, inflammatory bowel disease, lymphoma, cirrhosis, and coeliac disease. The liver signs suggest cirrhosis, and the Kayser-Fleischer ring suggests copper overload, as in Wilson’s disease. This is an autosomal recessive defect in copper metabolism, leading to cirrhosis, neurological sequelae, renal problems, and cardiac complications. Haemolytic anaemia can also occur due to copper deposits in RBC membrane.

How well did you know this?
1
Not at all
2
3
4
5
Perfectly
43
Q

An 8-year-old girl is brought to see you, having not opened her bowels in 8 days. She complains of hard painful stools and recurrent abdominal pain for the past 6 months, but no vomiting. Her mother thinks that she is avoiding going to the toilet and reports that she has always been a bit irregular opening her bowels, averaging about twice a week. In her past medical history, she passed meconium on day 1 of life, and has had no significant medical problems. On examination, she is a well-looking, normally grown child. Her abdomen is soft, with a palpable, indictable mass in the left iliac fossa. The anus is normal, as are her lower limbs. What is the first step in management?

A) Encourage her to increase her fluid intake, dietary fibre, and exercise
B) Introduce scheduled toileting with a positive reward scheme such as a star chart
C) Refer for bowel disimpaction under anaesthesia
D) Start polyethylene glycol with electrolytes such as Movicol
E) Start a stimulant laxative such as senna

A

A

Constipation is usually idiopathic, and can be treated conservatively in the absence of red flags such as growth failure, late meconium passage (?Hirschsprung’s disease), distension, anal pathology, or neurological complications. Once stools are softer, scheduled toileting can be introduced to help overcome avoidance.

If conservative treatment doesn’t work, medical treatment could be considered, including non-stimulant laxatives such as Movicol. Senna is not useful as it can cause more pain, dependence, and perforation.

Surgical referral would be a last resort.

How well did you know this?
1
Not at all
2
3
4
5
Perfectly
44
Q

A 2-week-old baby is brought to accident and emergency by his parents because he has been intermittently inconsolable for the past 12 hours. He does not want to breastfeed and has vomited. The parents think his tummy is upset as he keeps drawing up his legs. He was born at term by normal vaginal delivery with no problems. On examination, the abdomen is distended and tense. He is crying and there is a swelling in his right groin area. You can hear active bowel sounds. What is the most likely diagnosis?

A) Appendicitis
B) Right inguinal hernia
C) Gastroenteritis
D) NEC
E) Sepsis
A

B

These symptoms are consistent with an incarcerated inguinal hernia and requires urgent surgical intervention. All inguinal hernias in neonates require surgical review, but umbilical hernias self resolve.

Appendicitis is unusual in the very young as the neck of the appendix is wider. It’s more common in 11-20 years of age.

Gastroenteritis in young infants is usually a result of unsterilised water for making up formula.

NEC is predominantly in premature babies.

Sepsis is unlikely due to the lack of infectious sequelae.

How well did you know this?
1
Not at all
2
3
4
5
Perfectly
45
Q

A 5-year-old girl is brought to accident and emergency with a 24-hour history of vomiting and diarrhoea and now her eyes and skin have gone very yellow. She has been taking oral rehydration salts and is still passing urine. She is normally healthy and there is no family history of jaundice. On examination, her heart rate is 130bpm, and respiratory rate is 26. She is alert, warm, and well-perfused. The chest is clear, heart sounds are normal, and the abdomen is soft with a 2cm liver edge. What should the management be?

A) Reassure and discharge home, to return if not keeping fluids down
B) Take bloods to test for liver function, hepatitis, and urea and electrolytes; inform the Health Protection Agency, and discharge home with follow-up to review results
C) Take bloods to test for liver function, hepatitis screen, and urea and electrolytes, and admit for IV fluids
D) Take bloods to test for liver function, hepatitis screen, and urea and electrolytes, and admit for observation with continued oral rehydration salts
E) Take bloods to check liver function and urea and electrolytes. If they are normal, discharge home with reassurance, but to return if not keeping fluids down

A

B

Hepatitis A is a notifiable disease. She should not be admitted to reduce spread to other patients, and is currently well-hydrated so doesn’t require IV fluids.

How well did you know this?
1
Not at all
2
3
4
5
Perfectly
46
Q

A 15-year-old with well-controlled type 1 diabetes presents with frank haematemesis. Her blood tests in accident and emergency show: pH 7.37, glucose 18.3 mmol/L, haemoglobin 12.3 g/dL, white cell count 5.3x10^9/L, neutrophils 2.1x10^9/L, platelets 165x10^9/L, Na 135mmol/L, K 3.5mmol/L, urea 5.0mmol/L, creatinine 83 umol/L, alanine transferase 740 IU/L, bilirubin 96umol/L, alkaline phosphatase 102 IU/L, and albumin 25g/L. Further investigations once she is stable on the ward show hepatitis B surface antigen negative, anti-hepatitis C virus negative, anti-nuclear antibody (ANA) 1:320, and anti-smooth muscle antibodies are positive. What is the most likely diagnosis?

A) Autoimmune hepatitis with varices
B) Metabolic ketoacidosis
C) Gastroenteritis with a Mallory-Weiss tear
D) Pregnancy with hyperemesis and a Mallory-Weiss tear
E) Systemic lupus erythematosus (SLE)

A

A

T1D is most commonly associated with coeliac and Graves’ disease. Autoimmune hepatitis is more common in females and presents between 10 and 30 years of age. ANA is positive in 80% of cases and anti-smooth muscle in 70%. The raised glucose is a stress response.

How well did you know this?
1
Not at all
2
3
4
5
Perfectly
47
Q

An 18-month-old child is brought into accident and emergency with a 2-day history of vomiting, abdominal pain and fever. Which of the following is an unlikely cause of this clinical picture?

A) Lower lobe pneumonia with pain referred to the abdomen
B) Mesenteric adenitis
C) Diabetic ketoacidosis
D) Pyelonephritis
E) NEC
A

E

NEC is predominantly a disease of preterm infants in the neonatal period.

Children can present very generally. Respiratory disease can be associated with vomiting as inflamed upper airways trigger the gag reflex, and increased work of breathing increases intraabdominal pressure.

Mesenteric adenitis is most likely in this case, but is a diagnosis of exclusion. A viral illness causes enlarged mesenteric glands, observed on abdominal ultrasound.

DKA is uncommon but possible in chlidren this young.

Pyelonephritis is a common cause of fever and vomiting if no other clear source is identified.

How well did you know this?
1
Not at all
2
3
4
5
Perfectly
48
Q

A 13-month-old is referred up to her local district general accident and emergency by a GP who is concerned she has intussusception following an 18-hour history of fever, vomiting, and intermittent colicky screaming. A kind radiologist agreed to do an urgent ultrasound which shows an area of invaginated bowel in the right side of the colon. What is the most appropriate management?

A) Ask the radiologist to attempt a reduction by rectal air insufflation and if this fails make nil by mouth (NBM) and transfer to a local paediatric surgical unit
B) Make NBM and start IV fluids while waiting for transfer to a paediatric surgical unit
C) Move to theatre for an attempt of rectal air insufflation reduction and if this fails move to surgery in the local hospital as the patient will be too unstable for transfer
D) Make NBM, start IV fluids, and admit for observation
E) Make NBM and start IV fluids, and book him onto the emergency theatre list as he is too unstable for transfer to a local paediatric surgical unit

A

B

75% of cases can be reduced by rectal air insufflation, but failure has a risk of perforation so should be performed in a unit with the capability to operate. A DGH is likely unable to do this, so the child should be transferred.

How well did you know this?
1
Not at all
2
3
4
5
Perfectly
49
Q

Which of the following is not a cause of PR bleeding?

A) Constipation with an anal fissure
B) Intussusception
C) Meckel's diverticulum
D) Bacterial gastroenteritis
E) Abdominal migraine
A

E

Most common cause of PR bleeding is anal fissure. Intussusception has a late sign of redcurrant jelly stool, a mixture of blood and mucus from the distal invaginated segment becoming necrotic.
2% of the population have Meckel’s diverticulum, and it can ulcerate, perforate, and present with severe rectal haemorrhage of dark red blood.
Gastroenteritis with Shigella or Salmonella can produce blood mixed in the stool.

How well did you know this?
1
Not at all
2
3
4
5
Perfectly
50
Q

A mother brings her 2-year-old to see you. She is very worried that he always has diarrhoea or loose stools. He eats a normal diet, and no particular foods seem to upset him, but he often still has bits of vegetables or food he has eaten visible in the stool. She thinks he is losing weight, and he is starting to potty train, so she is concerned this will affect his ability to anticipate needing the toilet. On examination, he is an alert and well-looking child with a normal capillary refill, heart and respiratory rate. His abdomen is soft with no masses, there is no evidence of wasting, and his weight and height are following the 50th centile. What is the most appropriate management?

A) Reassure the mother, explaining this is toddler’s diarrhoea and he will grow out of it
B) Start loperamide as toddler’s diarrhoea is affecting his toilet training
C) Refer for endoscopy and biopsy to rule out coeliac disease
D) Refer for a colonoscopy and biopsy for inflammatory bowel disease
E) Order a blood test for thyroid function to rule out hyperthyroidism

A

A

This is toddler’s diarrhoea which is a common cause of loose stool in pre-school aged children and almost always requires no treatment. It is likely to be related to immature development of intestinal motility and it resolves in most children by the age of 5. Rarely, if the diarrhoea is socially disruptive to the child, it can be treated with loperamide.

How well did you know this?
1
Not at all
2
3
4
5
Perfectly
51
Q

A 10-year-old boy presents with recurrent mouth ulcers, abdominal pain, distension, and frequent episodes of diarrhoea with mucus. He has been losing weight. On examination, he is slim, and plotting his growth shows a fall in weight from the 50th centile to below the 9th. His abdomen is soft with generalised discomfort on deep palpation, but no masses are present. What is the most likely diagnosis?

A) Ulcerative colitis
B) Crohn's disease
C) Coeliac disease
D) Gastroenteritis
E) NEC
A

B

How well did you know this?
1
Not at all
2
3
4
5
Perfectly
52
Q

A 15-year-old boy comes to see you, complaining of recurrent abdominal and back passage pain, relieved by passage of diarrhoea. He is also complaining of low back and knee pain, and last week there was blood mixed into his stool. He has been losing weight recently. On examination, he is slim, and looks pale. His abdomen is soft, but tender in the left iliac fossa, with no masses. What is the most likely diagnosis?

A) Ulcerative colitis
B) Crohn's disease
C) Coeliac disease
D) Gastroenteritis
E) NEC
A

A

How well did you know this?
1
Not at all
2
3
4
5
Perfectly
53
Q

99% of healthy term infants will pass meconium within the first 24h of life, and all should do so within 48 hours. Which of the following is not a cause of delayed meconium passage?

A) Hirschsprung's disease
B) Cystic fibrosis with a meconium ileus
C) Choanal atresia
D) Imperforate anus
E) Meconium plug syndrome
A

C

How well did you know this?
1
Not at all
2
3
4
5
Perfectly
54
Q

A 3-month-old baby is brought to accident and emergency because he has been vomiting and having diarrhoea for the past month. His mother breastfed him until he was 8 weeks old, and he is now taking formula milk, 4-5oz every 4 hours. On examination, he is alert, but fussy, and looks thin. He has eczema on his face, neck, and torso, and the mother says this is new. The abdomen is soft, the genitalia are normal with a significant nappy rash, and the anal margin is erythematous. You plot his growth in his red book, and find that he was born on the 50th centile, and was following that, but now is on the 25th centile for weight. What is the most likely diagnosis?

A) Cow's milk protein intolerance
B) Lactose intolerance
C) Gastroenteritis
D) Hyper IgE syndrome
E) Wiskott-Aldrich syndrome
A

A

Cow’s milk protein intolerance presents in infancy with skin inflammation, vomiting, diarrhoea (which may be bloody), failure to thrive, irritability, and colic.

Lactose intolerance is typically acquired after acute gastroenteritis, with a deficiency in lactase.

The history is too long for gastroenteritis.

Hyper IgE syndrome is an autosomal dominant immunodeficiency associated with severe eczema and skin boils.

Wiskott-Aldrich syndrome is an X-linked recessive disorder associated with thrombocytopenia, eczema, and lymphopenia.

How well did you know this?
1
Not at all
2
3
4
5
Perfectly
55
Q

A 15-year-old Asian girl with Down’s syndrome came to accident and emergency with a prolonged fever. She has severe learning difficulties and was difficult to assess. Her parents think she is more unsettled than usual, and not eating and drinking properly for the last 3 weeks. She is admitted as you cannot confidently find the source of the infection, but she has no cough, rash, vomiting, diarrhoea, or meningism. The next day, she complains of a headache and starts to vomit. She has a CT scan which is normal, and then a lumbar puncture (LP). WCC 150x10^9/L (20% neutrophils), RBC 0, protein 2g/L, glucose 1.2mmol/L (serum glucose 6.0mmol/L). What is the most likely cause of this meningitis?

A) Mycobacterium tuberculosis
B) Herpes simplex virus
C) Streptococcus pneumoniae
D) Cryptococcus neoformans
E) Neisseria meningitidis
A

A

Insidious onset and lack of meningism in this case suggests that the meningitis is caused by TB.

Streptococcus pneumoniae and Neisseria meningitidis are the most common causes of meningitis in the UK. They cause raised white cells, normal protein, and glucose less than 2/3 of the serum glucose.

HSV causes encephalitis, which is clinically similar to bacterial meningitis. HSV PCR can be performed on CSF from the LP.

Cryptococcus neoformans is more likely in the immunosuppressed patient.

How well did you know this?
1
Not at all
2
3
4
5
Perfectly
56
Q

You are on elective in Uganda and spending the day on the paediatric ward. You are told that it is the rainy season and malaria is now becoming increasingly problematic. Almost all the children on the ward are suffering with the effects of malaria. The first child is a 5-year-old boy with a cyclical fever, abdominal pain, and a 4cm splenomegaly. He has 2% parasitaemia on blood film. you are asked how you would treat this child. What is the best initial management step?

A) IM quinine
B) IV fluids and IV quinine
C) IV fluids and prophylactic splenectomy
D) Emergency splenectomy
E) Oral atovaquone
A

B

Anything above 2% is a moderate parasitaemia. Regardless, malaria requires hospitalisation and quinine (PO if uncomplicated, but IV dependent on severity or complications).

Splenectomies are only indicated if spleen is ruptured (e.g. hypovolaemic shock, peritonism).

Antimalarials should be taken dependent on local resistance of strains. Advice should be given regarding prophylaxis in the future, including chemopropylaxis, and also measures such as insect repellents and mosquito nets.

How well did you know this?
1
Not at all
2
3
4
5
Perfectly
57
Q

A 3-year-old girl presents to accident and emergency with a 6-day history of fever and she is over 38 degrees Celsius when measure by her mother with a tympanic thermometer. She has become very miserable for the last few days. She has developed a rash on her trunk, which is blanching, erythematous and confluent. On examination, you also note bilateral non-purulent conjunctivitis, cervical lymphadenopathy, and a red tongue with lip cracking. Her extremities are also erythematous but not peeling. WCC 14x10^9/L, CRP 200mg/L, and ESR 60 mm/hour. Blood culture is pending. What is the diagnosis?

A) Staphylococcal scalded skin
B) Toxic shock syndrome
C) Scarlet fever
D) Kawasaki's disease
E) Measles
A

D

This is Kawasaki disease. A diagnosis requires at least a 5d history of fever, with 4 of:
Bilateral non-purulent conjunctivitis
Rash
Red/cracked lips with strawberry tongue
Erythematous/peeling/oedematous extremities
Cervical lymphadenopathy
Kawasaki disease is a small-medium vessel vasculitis of unknown aetiology, in keeping with the raised ESR. It requires treatment with IVIG and aspirin. There is a risk of coronary artery aneurysm if untreated.

Staphylococcal scalded skin causes peeling with fever.

Toxic shock syndrome causes rash, fever, and diarrhoea.

Scarlet fever gives a sandpaper rash with erythematous mucous membranes.

Measles causes fever and an erythematous macular rash.

How well did you know this?
1
Not at all
2
3
4
5
Perfectly
58
Q

A 3-year-old boy presents with a right swollen eyelid. He has had a cold for the last week, but his eyelid started swelling yesterday. He has had no injury or broken skin around the eye. On examination, his right eye is swollen and red, there is no discharge, he is now unable to open his right eye, and he has proptosis. You are concerned about the complications of this infection. Within the last hour he has become drowsy and started to vomit. His observations are all normal. What is the concerning complication in this case?

A) Visual loss
B) Abscess
C) Septicaemia
D) Orbital cellulitis
E) Meningitis
A

E

All of those listed are a complication of periorbital cellulitis. The reduced consciousness and vomiting would raise concern of meningitis, though.

How well did you know this?
1
Not at all
2
3
4
5
Perfectly
59
Q

A 14-year-old girl presented to the GP with an enlarged lymph node in her neck. She first noticed it 3 weeks ago, and it is increasing in size. She has had a dry cough, fevers, night sweats, and weight loss. She has had a poor appetite over the last 2 weeks, which her mother blames for her weight loss. There is no history of foreign travel or tuberculosis (TB) contacts. A CXR shows a mediastinal mass. What is the most likely diagnosis?

A) Lymphoma
B) Pneumonia
C) TB
D) Lung tumour
E) Leukaemia
A

A

Prolonged fever raises the possibility of non-infectious causes. The ‘B’ symptoms and rapidly enlarging lympadenopathy suggest lymphoma. Leukaemia can also cause lymphaenopathy, but is more likely to present with anaemia, infections, or bleeding.

Pneumonia isn’t seen on the CXR, so is unlikely.

Lack of TB contacts and foreign travel make this unlikely.

Lung tumours are rare in children.

How well did you know this?
1
Not at all
2
3
4
5
Perfectly
60
Q

A 6-year-old girl presents to accident and emergency with a fever. She has no history of cough, cold, vomiting, diarrhoea, rash, headache, or joint pain. On examination, she is tachycardic at 150bpm, and there are two petechial spots on her right ankle. Her capillary refill time is 4 seconds, and she has cold feet. All her other observations are normal. What is the most appropriate course of action?

A) Inform the consultant about child protection concerns
B) IV ceftriaxone
C) IV fluid bolus and IV ceftriaxone
D) Admit to the ward for observation
E) Discharge
A

C

This girl is tachycardic and therefore should be investigated to find the cause. With the petechial spots in the presence of a fever and clinical signs of shock, one must consider meningococcal sepsis and commence antibiotics immediately. Since she has features of early shock, namely
tachycardia and prolonged capillary refill time, she therefore requires a fluid bolus so (C) is correct, not (B).

How well did you know this?
1
Not at all
2
3
4
5
Perfectly
61
Q

A 3-year-old boy was brought to accident and emergency with his mother. She says he has been limping for a day now and refusing to walk for the last 2 hours. He has had a fever to 39 degrees Celsius, which can be brought down with paracetamol. He has had no vomiting, diarrhoea, rash, cough, coryza, or injury. He lives with his mother and is her only child. She is currently unemployed and has a background of depression. On examination of the right leg he has a swollen thigh and cries inconsolably when it is touched. It is red and tender. He refuses to allow movement of the hip either passive or active. The left leg is unremarkable on examination. What is the most likely diagnosis

A) Perthes’ disease
B) Septic arthritis
C) Fractured femur due to accidental injury
D) Juvenile idiopathic arthritis
E) Fractured femur due to non-accidental injury

A

B

The fever, limp, limited movement, and signs of acute inflammation suggest septic arthritis.

Perthes’ disease usually presents as intermittent pain and limp between the ages of 5 and 7, so is unlikely here.

The lack of trauma history makes fracture unlikely.

JIA requires 6 weeks of joint pain and swelling persisting after other diagnoses have been excluded.

How well did you know this?
1
Not at all
2
3
4
5
Perfectly
62
Q

A pregnant woman attends her booking appointment at the antenatal clinic and has her routine blood tests done. She is now 13 weeks pregnant with her first child and you have a positive result for cytomegalovirus (CMV) IgM. You need to discuss the implications of CMV infection on her unborn child. Which of the following are not features of congenital CMV infection?

A) Deafness
B) IUGR
C) Hydrocephalus
D) Thrombocytopenia
E) Congenital cardiac defects
A

E

Congenital CMV can be transmitted in utero, and can result in IUGR, neurological sequelae (such as hydrocephalus and sensorineural deafness), and thrombocytopenia.

Congenital cardiac defects are more associated with maternal rubella infections.

How well did you know this?
1
Not at all
2
3
4
5
Perfectly
63
Q

A 5-year-old girl was admitted to the ward after she presented to her local accident and emergency with diarrhoea. She was passing 7-8 loose, watery stools per day for the last 4 days and had been vomiting for 1 day prior to this. There was blood in the stools and this had worried her mother. You ask about foreign travel and her mother reveals they had been in India until 2 weeks ago, staying with family and drinking tap water. She had no vaccines prior to travelling. On examination, she now has abdominal pain, swinging pyrexial, right upper quadrant tenderness, but no rebound or guarding. You notice a pale pink (rose) spot on her trunk. What is the most likely infecting organism?

A) Rotavirus
B) Shigella spp.
C) Vibrio cholera
D) Salmonella typhi
E) Escherichia coli 0157
A

D

Rotavirus is the most common cause of infective gastroenteritis worldwide, but blood in the stool suggests a bacterial cause, and the rose spot suggests Salmonella typhi. This is a notifiable disease.

Shigella and E. coli would cause all of the symptoms except the rose spot.

Cholera would present with dysentery, not just diarrhoea. This is a notifiable disease.

How well did you know this?
1
Not at all
2
3
4
5
Perfectly
64
Q

A 10-month-old baby is brought to accident and emergency by ambulance having had a seizure. His mother reports that he went floppy suddenly and then his right arm and leg started shaking and he was not crying. It lasted less than 5 minutes and he was sleepy afterwards. He has had a fever and runny nose for the last 2 days and is off his food. Why is this not a febrile seizure?

A) He is too young
B) He has had a focal seizure
C) He has recently had a viral illness
D) The seizure lasted too long
E) The fever was not high enough
A

B

Febrile seizures are common from 6m to 6y. They mostly occur in viral illnesses, and are generalised seizure in association with any fever (>37.5).

A focal seizure like this, with infection, raises concerns of meningitis.

How well did you know this?
1
Not at all
2
3
4
5
Perfectly
65
Q

A

A 4-year-old boy has been brought into accident and emergency with breathing problems. He is assessed by the paediatric team and found to have inspiratory and expiratory stridor, audible wheeze, lip and tongue swelling, and an urticarial rash on his trunk and abdomen. His heart rate is 167bpm and his respiratory rate is 40. BP is 90/45mmHg. What is the single most important management step?

A) Do not examine his throat as this may distress him
B) Give a normal saline fluid bolus
C) Give IV adrenaline 0.1mg/kg of 1:10 000
D) Give IM adrenaline 0.01 mg/kg of 1:1000
E) Mobilise the paediatric anaesthetist as his airway is compromised

A

D

This is anaphylactic shock. First, an IM injection of 1:1000 adrenaline should be given. Then an ABC approach including anaesthetist involvement for the airways, and fluid bolus for circulation, can be taken.

How well did you know this?
1
Not at all
2
3
4
5
Perfectly
66
Q

A 12-year-old girl with a history of discitis in her lumbar spine was admitted following investigation at her tertiary centre. She was started on IV benzylpenicillin and clindamycin. She received 24 hours of medication and a rash appeared on her trunk and arms. There were discrete red lesions which outlined a central target lesion. They were non-blanching and itchy. What is the most likely diagnosis?

A) Erythema migrans
B) Erythema toxicum
C) Erythema marginatum
D) Erythema nodosum
E) Erythema multiforme
A

E

Erythema multiforme is a target rash caused by medications or infections, and also can be idiopathic.

Erythema migrans is a gradually expanding red patch from the site of a tick signifying Lyme disease.

Erythema toxicum is a benign maculopapular rash found in neonates.

Erythema marginatum is an annular erythema seen in children with rheumatic fever, alongside carditis, arthralgia, subcutaneous nodules, and Sydenham’s chorea.

Erythema nodosum is usually caused by IBD, but can also be triggered by medications, infection, autoimmune disease, and malignancy. It is a erythematous rash of the anterior calves.

67
Q

A 3-year-old boy is admitted to the children’s ward. He has been isolated in a cubicle as he is at risk of infections. He is awaiting a bone marrow transplant and has a brother with the same condition. His mother tells you they both have SCID. What are the likely immune function test results in SCID?

A) Normal B cells, normal T cells, normal immunoglobulins
B) Low B cells, low T cells, low immunoglobulin
C) Normal B cells, normal T cells, high immunoglobulin M subsets
D) Low B cells, normal T cells, normal immunoglobulins
E) Normal B cells, low T cells, normal immunoglobulins

A

B

Severe combined immunodeficiency results in a reduction in both B and T cell lines, therefore causing poor cellular and humoural immunity. Bone marrow transplant is usually curative.

68
Q

A 3-week-old baby attends accident and emergency with bloody diarrhoea. Mum says he has been having diarrhoea for the past 2 days since she started using formula milk. He was previously breastfed and mum was not having any dairy products due to lactose intolerance. He also has eczema on his cheeks and a strong family history or asthma and eczema. Mum is concerned that he may be allergic to milk too. What is the most likely diagnosis?

A) Lactose intolerance
B) Gastroenteritis
C) Cow's milk protein intolerance
D) Fructose intolerance
E) Galactosaemia
A

C

Cow’s milk protein intolerance presents with diarrhoea on introduction
of cow’s milk based formula or breast milk if the mother takes dairy products. It may be associated with eczema, commonly on the face.

Lactose intolerance can present similarly, but the congenital form is extremely rare. Lactase tends to reduce from 2 years of age in most people.

Fever is not present, so gastroenteritis is unlikely.

Fructose intolerance is a rare metabolic condition resenting with vomiting, hypoglycaemia, failure to thrive, hepatomegaly, jaundice, renal complications and severe metabolic acidosis.

Galactosaemia is due to galactose-1-phosphate uridyl transferase deficiency, which can present with metabolic disturbances, sepsis, vomiting and collapse.

69
Q

A 2-year-old child is brought to cardiology clinic due to a heart murmur heard by the GP after an examination when she was recently unwell. She was born at 40 weeks by normal vaginal delivery, but was noted to have a cleft palate at birth. She was kept in hospital for establishment of feeding, but during this time she had a seizure, noted later to be due to hypocalcaemia. You hear a harsh, grade 3/6 pan systolic murmur, loudest at the left lower sternal edge, consistent with a ventral septal defect (VSD) as seen on echocardiogram. With this history and current examination finding, you wish to exclude DiGeorge’s syndrome. What is the best diagnostic test?

A) Karyotype
B) FISH (fluorescence in situ hybridisation)
C) ELISA (enzyme-linked immunosorbent assay)
D) Geneticist review and diagnosis
E) Identification of specific mutation

A

B

DiGeorge syndrome is due to a microdeletion on chromosome 22q11.2. It manifests as a collection of features including cleft palate, aortic arch and other cardiac abnormalities, thymic hypoplasia, typical facial features and hypocalcaemia. Microdeletions are best detected with FISH.

70
Q

A couple are referred to a geneticist as they are planning on having their first child. There is a history of Wiskott-Aldrich syndrome on the woman’s side. The woman’s father and great grandfather have the condition (eczema, thrombocytopenia, recurrent infection), but she is unaffected. There is no history of the condition in the man’s family. What is the risk of having the condition if the child is a boy or a girl respectively?

A) Boy: 1/4; Girl: 1/4
B) Boy: 1/2; Girl: 0
C) Boy: 0; Girl: 0
D) Boy: 0; Girl: 1/2
E) Boy: 1; Girl: 0
A

B

Wiskott-Aldrich syndrome is an X-linked condition. If the woman’s father is a sufferer, but she is not, the woman is a carrier. She has a 1/2 chance of passing the affected chromosome onto her child. If the child is a girl, she will receive a non-affected X chromosome from the father, so will be a carrier or unaffected. If the child is a boy, he will receive a Y chromosome, and therefore has a 1/2 chance of being affected.

71
Q

A 4-year-old boy with severe eczema is brought to accident and emergency by his mother. His skin has been worse recently since the weather has become colder. He is scratching a lot more and now is very miserable and has a temperature of 38.6 degrees Celsius today. On examination of his skin, he has multiple areas of erythematous, excoriated lesions on his elbow and knee flexures as well as his trunk and back. In addition they are hot, tender, and slightly swollen with areas of broken skin. There are also some yellow fluid-filled vesicles on some of these lesions. You send some blood tests, and commence him on IV flucloxacillin and acyclovir. Which are the two most likely organisms that can complicated eczema?

A) Gram-positive cocci and herpes simplex virus (HSV)
B) Gram-negative cocci and HSV
C) Gram-positive cocci and varicella zoster
D) Gram-negative bacilli and herpes zoster
E) Gram-positive bacilli and HSV

A

A

Cellulitis and eczema herpeticum are the two important complications. Gram positive cocci are responsible for cellulitis, as they are commensal on the skin. It requires aggressive treatment to protect against sepsis.

72
Q

A 3-year-old boy has been admitted to hospital with a right-sided pneumonia and pleural effusion. The pleural fluid grew Gram-positive cocci. He is on IV ceftriaxone, oral azithromycin, and has a chest drain in situ. On further questioning of the boy’s mother, you establish that he has had multiple chest infections since he was born (in the UK). He has been admitted three times before and also had a sinus wash out following an episode of sinusitis. He has no cardiac anomalies or dysmorphism. His mother also tells you about his older brother, who sadly died of meningitis aged 6 years old. He too had ‘more than his fair share of infections’. The two brothers had different fathers, and his mother is HIV negative. What is the most likely underlying immunodeficiency in this family?

A) DiGeorge's syndrome
B) Complement deficiency
C) X-linked agammglobulinaemia
D) Subacute combined immunodeficiency disorder (SCID)
E) HIV
A

C

73
Q

You are in immunology clinic and the first patient is a 2-year-old boy who has a complement deficiency. You know this involves a cascade of proteins involved with innate immunity, but are unsure about the manifestations in children. The professor of immunology asks you which organism is this child at risk of being infected with. He gives you a clue by telling you the child has a late complement deficiency, meaning C5-C9. What is the most likely causative organism that infects these children?

A) Streptococcus pneumoniae
B) Neisseria meningitidis
C) Haemophilus influenza
D) Mycobacterium tuberculosis
E) Pneumocystis jiroveci
A

B

Deficiencies in protein C3 resulting in recurrent pyogenic infection, protein C1, C2 and C4 associated with autoimmunity and proteins C5–9 with a risk of getting infections with N. meningitidis.

74
Q

A 13-year-old girl is on the ward having a bone marrow transplant for acute leukaemia. She is noted to be profoundly anaemic, with a Hb of 5.9 g/dL. She is due to receive a transfusion of one unit of RBCs. You are called to see her 5 minutes after starting the transfusion. She has come out in a rash, and is looking frightened. She has a HR of 120bpm and RR of 30. As you arrive, you can see she has swollen lips and tongue, and her BP is 90/45mmHg. What is the best initial management step?

A) Repeat a full set of observations as it is likely to be anxiety resulting in the abnormal HR, RR, and BP. If still abnormal, stop transfusion
B) Stop the transfusion and return the unit to the blood bank
C) Stop the transfusion, take down the giving set, give IM adrenaline immediately
D) Give IM adrenaline, stop the transfusion, take down the giving set
E) Stop the transfusion, give IM adrenaline immediately, and restart if the reaction settles

A

C

This is anaphylaxis. First, the allergen should be removed, so the transfusion should be stopped and the giving set taken down. Then IM adrenaline can be given for the anaphylactic shock.

75
Q

A 5-year-old boy was diagnosed with asthma aged 3 years. He presented to accident and emergency with shortness of breath, increased work of breathing, and a 1-week history of coryzal illness and fever. On examination, he is tachypnoeic 60/min, tachycardic 160 bpm, and has minimal air entry bilaterally. He has intercostal recession, tracheal tug, and is too breathless to complete a sentence. Oxygen saturation is 90% in air. What is the initial management of this boy?

A) Immediate intubation and ventilation
B) High flow oxygen through non-rebreather mask
C) IV salbutamol and magnesium sulphate infusion
D) Back to back salbutamol and Atrovent through oxygen driven nebuliser
E) Trial of continuous positive airway pressure support

A

D

British Thoracic Society guidelines give a stepwise approach to chronic and acute asthma. This is life-threatening asthma based on their classification, but initial management is always back-to-back salbutamol nebulisers. After 3 doses of salbutamol and 1 of Atrovent, he may then require IV salbutamol or aminophylline with magnesium sulphate, but this is not first line.

76
Q

A 3-year-old child presents to the GP with a chronic cough for the last month. He had previously been fit and well since he suffered a severe pertussis infection when he was 1 month of age. He has subsequently been fully immunised but was noted to be on the 0.4th centile for height. What is the most likely cause for his cough?

A) Cystic fibrosis
B) Recurrent pertussis infection
C) Habit
D) Asthma
E) Bronchiectasis
A

E

A severe respiratory infection in childhood can present later with bronciectasis. This is dilatation and poor mucociliary clearance, predisposing to further infection.

77
Q

A 15-year-old boy attends his GP with a week of cough productive of yellow sputum, fever to 39 degrees Celsius and chest pain on the right side of the chest on coughing. There is no history of foreign travel or unwell contacts. On examination there is reduced air entry in the right lower zone with crepitations and bronchial breathing. You diagnose a right-sided chest infection. What is the most likely causative organism?

A) Staphylococcus aureus
B) Mycobacterium tuberculosis
C) Streptococcus pneumoniae
D) Mycoplasma pneumoniae
E) Chlamydophila pneumoniae
A

D

Pneumonia organisms can be typical (S. pneumoniae, S. aureus, H. influenzae, M. catarrhalis) or atypical (M. pneumoniae, Legionella, Chlamydia).

S. pneumoniae is most common in children under 4, but M. pneumoniae is most common in older children.

78
Q

A 4-year-old girl has recently moved to the area and is registering with you, her new GP. She has had a diagnosis of primary ciliary dyskinesia (PCD) made last week and the parents wish to know more about the complications. Which of the following is not a complication of PCD?

A) Pancreatic insufficiency
B) Infertility
C) Sinusitis
D) Bronchiectasis
E) Dextrocardia
A

A

Pancreatic secretions are not reliant on cilia.

The respiratory tract and reproductive organs are lined with cilia, and cilia are also necessary for determination of sites of internal organ development, so children may have dextrocardia, or situs inversus.

79
Q

A 26-week premature baby was born by emergency C section due to maternal pre-eclampsia. He required ventilation until age 38 weeks corrected gestation, and is still requiring oxygen to maintain his saturations. At 12 months, he has poor vision, and neurodevelopment function, requires home oxygen and was admitted for a recent respiratory syncytial virus (RSV) bronchiolitis. What is the underlying diagnosis of his respiratory problems?

A) Respiratory distress syndrome (hyaline membrane disease)
B) Chronic lung disease (bronchopulmonary dysplasia)
C) Cystic fibrosis
D) Diaphragmatic hernia
E) Pulmonary hypoplasia

A

B

Chronic lung disease is an oxygen requirement at 36 weeks corrected gestation or at 28d post-term. There can be long-term consequences such as RSV.

RDS is surfactant deficiency, requiring intubation and exogenous surfactant. This can develop to CLD.

CF doesn’t present as lung disease neonatally. More likely is meconium ileus or prolonged jaundice.

Diaphragmatic hernia and renal abnormalities can cause pulmonary hypoplasia due to the lungs not having enough space to develop, and presents as respiratory difficulties at birth.

80
Q

A 2-year-old boy is brought in whose mother has been concerned about a cough for the last 2 weeks which started out of the blue. He has been previously fit and well with no respiratory or cardiac problems from birth. There is no family history of illness. He is thriving and eating as normal, but has a persistent cough, recently productive of yellow and slight blood-stained sputum. You suspect that he may have a pneumonia and lung collapse secondary to an inhaled foreign body. Which is the most likely location of this boy’s foreign body?

A) Left lower lobe
B) Right upper lobe
C) Right middle lobe
D) Left upper lobe
E) Right lower lobe
A

C

The right main bronchus is the shortest, widest, and most vertical bronchi, with the middle lobe being the most likely terminal.

81
Q

A couple who are known to both be carriers of cystic fibrosis ask to see you. They had genetic counselling but declined antenatal diagnostic testing and their baby has now been born and is ready to be discharged home. The parents are now keen to get the baby tested so that if treatment is required it can be initiated early on. What initial test do you suggest for the baby?

A) Newborn blood spot screening
B) Chest X-ray
C) Faecal elastase
D) Genetic testing
E) Sweat test
A

A

The newborn blood spot screen is ideally performed at 5 days, and tests for 9 diseases:

Cystic fibrosis
Congenital hypothyroidism
Sickle cell disease
Phenylketonuria
Maple syrup urine disease
Medium chain acyl dehydrogenase
Isovaleric acidaemia
Glutaric aciduria type 1
Homocystinuria

The last 6 are inherited metabolic disorders.

82
Q

A 5-week-old baby was admitted today to the children’s ward with bronchiolitis. The nasopharyngeal aspirate identified respiratory syncytial virus. He was saturating to 96% in air this morning and was feeding two-thirds of his usual amount of formula milk. You are asked to review him as his work of breathing is worsening now it is night time. He has nasal flaring, intercostal and subcostal recession, tachypnoea, and crepitations and wheeze heard bilaterally. What do you expect his capillary blood gas to show?

A) pH 7.16; PCO2 3.1kPa; PO2 10.0kPa; BE -8; HCO3 18mmol/L
B) pH 7.38; PCO2 5.5kPa; PO2 12.0kPa; BE +1; HCO3 25mmol/L
C) pH 7.20; PCO2 8.2kPa; PO2 8.3kPa; BE +2; HCO3 26mmol/L
D) pH 7.40; PCO2 1.2kPa; PO2 7.5kPa; BE +5; HCO3 28mmol/L
E) pH 7.47; PCO2 6.3kPa; PO2 11.0kPa; BE +10; HCO3 35mmol/L

A

C

C is type 2 respiratory failure with hypercapnia and hypoxia. This is due to his worsening worth of breathing, as he is tiring and failing to oxygenate effectively.

Before tiring, he is likely to have had a respiratory alkalosis with metabolic compensation, as in D.

83
Q

A 14-year-old girl with muscular dystrophy now mobilises in a wheelchair and other co-morbidities include a scoliosis and cardiomyopathy. She is being seen for her annual review in clinic. Which of these would best represent the respiratory complications of muscular dystrophy?

A) Normal FVC, low FEV1/FVC ratio
B) Flattened diaphragm on chest x-ray
C) Morning dips in peak expiratory flow rate
D) Extrathoracic obstruction on flow-volume loops
E) Reduced FVC, normal FEV1/FVC ratio

A

E

Muscular dystrophy causes restrictive disease, with reduced FVC and normal FEV1/FVC ratio.

Normal FVC with reduced FEV1/FVC is an obstructive pattern seen in asthma, as are morning dips in PEF.

Diaphragmatic flattening is seen in hyperinflation, such as in bronchiolitis.

Extrathoracic obstruction would give symptoms of chest pain, cough, or lymphadenopathy.

84
Q

A 10-month-old baby boy is brought to accident and emergency with inconsolable crying. His mother says he is a miserable baby and even after feeding does not settle. He has recently started to cruise around furniture, but is not yet walking. His crying has been worse today and both his parents had been awake all night due to his incessant crying. On examining the baby, you not that he is more upset when being handled, and is a bit better when lying on his front. You do a chest x-ray which shows three posterior rib fractures; his mother states he fell down some steps yesterday. What is the likely diagnosis and appropriate management strategy?

A) Birth trauma; no intervention necessary as they will heal spontaneously
B) Accidental injury; ensure no pneumothorax present and admit for observation
C) Accidental injury; ensure no pneumothorax present and admit for observation
D) Non-accidental injury (NAI); advise the parents you will refer to social services and discharge home
E) NAI; discuss with social services and paediatric consultant and admit the child to a place of safety

A

E

Posterior rib fractures are highly suspicious of NAI. The child is 10m old, so the injuries are not consistent with development or the proposed mechanism. If you suspect NAI, then a detailed history and examination are needed, discussion with a senior paediatrician and social services to fully investigate the situation to safeguard the child.

85
Q

What is the most common congenital heart defect?

A) Coarctation of the aorta
B) Ventricular septal defect (VSD)
C) Atrial septal defect
D) Patent ductus arteriosus
E) Transposition of the great arteries
A

B

1/3 congenital heart defects are VSDs. The second most common is PDA, with just over 10%. ASDs, CoA, and TGA make up 5% each.

86
Q

Which of the following is not a presenting symptom or sign associated with congenital heart disease?

A) Respiratory distress with feeds
B) Cyanosis
C) Hepatosplenomegaly
D) Vomiting with feeds
E) Sweating with feeds
A

D

Respiratory distress and sweating with feeds is a classic presentation of congenital cardiac disease.

Cyanosis is present in Tetralogy of Fallot and transposition of the great arteries.

Hepatosplenomegaly can be a result of heart failure.

Vomiting with feeds is not usually associated with heart disease, more likely with gastro-oesophageal reflux or pyloric stenosis.

87
Q

Which of the following is not a feature of an innocent murmur?

A) Systolic murmur
B) Diastolic murmur
C) Asymptomatic
D) Heard only at the left sternal edge
E) No heaves or thrills
A

B

Innocent murmurs are soft-blowing, systolic, heard at the left sternal edge in asymptomatic patients with no radiation, no diastolic component, and no parasternal thrill.

88
Q

You are asked to see a 2-year-old child with difficulty in breathing, a runny nose, and a barking cough. His mother tells you he had a heart defect repaired as a baby and he still has a murmur. On examination, he has noisy breathing with mild subcostal recession. He is apyrexial, with an RR of 44/min, and HR of 152bpm. Capillary refill is 1-2 seconds. His throat is red and the tonsils are enlarged with no exudate. On his chest you see a midline sternotomy scar with a drain scar and a right thoracotomy scar. On auscultation, the lung fields are clear, but he has an ejection systolic murmur in the left upper sternal edges which radiates to the back. He does not have a gallop rhythm. There are transmitted upper airway sounds only on the lung fields and the abdomen is soft with no organomegaly. What is the most appropriate management?

A) Admit for IV antibiotics
B) Give IV furosemide and admit
C) Admit for observation
D) Send home on oral antibiotics
E) Give oral dexamethasone and observe
A

E

This child has had corrective cardiac surgery and has a residual pulmonary stenosis murmur.

He likely has croup. This can be mild, moderate, or severe. In this case, he has mild recessions, stridor at rest, markedly decreased air entry. This scores 5 on the Westley score, putting him in the moderate category. He should be admitted and treated with dexamethasone. It should improve over the next few days.

89
Q

A 3-day-old baby is brought to accident and emergency with acute respiratory distress. She is tachypnoeic, tachycardic, cyanosed, and her capillary refill is 5 seconds centrally. You note she has a flat nasal bridge, down sloping palpebral fissures and epicanthic folds. On auscultation, there is a loud ejection systolic murmur at the left sternal edge. What is the most likely diagnosis?

A) Coarctation of the aorta
B) VSD
C) Transposition of the great arteries
D) Tetralogy of Fallot
E) Patent ductus arteriosus
A

D

This baby has cyanotic heart disease. This means the answer is not coarctation, VSD, or PDA.

Cyanotic heart disease is duct-dependent, so she requires urgent prostaglandins to keep the duct open until she can have corrective cardiac surgery.

As she also has Down syndrome, it is most likely that this is tetralogy of Fallot, as that, alongside ASDs and VSDs are associated with T21.

90
Q

You are doing a baby check on the post-natal ward on a baby who is 23 hours old. His mother tells you that he is not feeding well. On examination he is unsettled with an RR of 76 and a HR of 182. You think his hands and feet look blue, and there is a soft systolic murmur heard at the left upper sternal border. You ask the midwives to check his saturations which are 85% in air, and you start oxygen. You explain to the mother that he needs to be managed on the neonatal unit. What is the next step in your management?

A) Stop the oxygen as this may drive the closure of the ductus arterioles
B) Give prostaglandin intravenously to open the duct while organising an echocardiogram
C) Give antibiotics and prostaglandin intravenously while organising an echocardiogram
D) Give indomethacin intravenously to open the duct while organising an echocardiogram
E) Give indomethacin and antibiotics intravenously while organising an echocardiogram

A

C

This baby has suspected congenital cyanotic heart disease, which are duct-dependent. Thus prostaglandins should be given to maintain patency.

Sepsis could also present with mottling, flow murmur, tachycardia, and tachypnoea, so Abx should be given to cover this, while an echocardiogram can confirm the diagnosis.

91
Q

A 7-year-old with a 3-day history of upper respiratory injection is brought to accident and emergency by his mother because he suddenly went pale and sweaty and seems to be working hard to breath. The triage nurse calls you to see him urgently because his heart rate is 200bpm. You take him round to the resuscitation area, give him oxygen and connect him to the cardiac monitor. The ECG shows a narrow complex tachycardia with a rate of 180bpm. He remains alert, with a respiratory rate of 40. What is the most appropriate initial diagnosis?

A) Supraventricular tachycardia (SVT)
B) Wolff-Parkinson-White syndrome
C) Ventricular fibrillation
D) Atrial fibrillation
E) Ventricular tachycardia
A

A

A narrow complex tachycardia of over 180bpm suggests SVT.

This may be triggered by Wolff-Parkinson-White syndrome, but this can only be diagnosed by a delta wave in all QRS complexes once the rhythm has slowed.

Ventricular fibrillation is an irregular wide complex tachycardia with no discernible P waves.

Ventricular tachycardia is a regular, wide complex tachycardia with no P waves.

Atrial fibrillation has a saw toothed P wave baseline with slower QRS complexes superimposed.

92
Q

A 7-year-old with a 3-day history of upper respiratory infection is brought to accident and emergency by his mother because he suddenly went pale and sweaty and seems to be working hard to breath. The triage nurse calls you to see him urgently because his heart rate is 200bpm. You take him round to the resuscitation area, give him oxygen and connect him to the cardiac monitor. The ECG shows a narrow complex tachycardia with a rate of 180bpm. He remains alert, with a respiratory rate of 40. What is the first step in management?

A) Non-synchronised shock
B) Adenosine
C) Adrenaline
D) Vagal manouevres
E) Synchronised shock
A

D

If the child is still alert, vagal manoeuvres would be the first step, such as asking a child to blow into a syringe like a balloon, unilateral carotid massage, or putting the head in ice. These should be carried out with continuous ECG.

If conservative measures don’t work, or the child is not alert, medical cardioversion is indicated, with adenosine.

If there is still no response, sedated synchronised cardioversion is next.

Non-synchronised shock is only indicated in VF or VT with no cardiac output.

Adrenaline would only be indicated if the child loses output.

93
Q

A 2-year-old child is referred to hospital by the GP after his third visit that week. He now has a rash, and the GP is worried that he has meningitis. He has had a fever for 5 days up to 39.5 degrees Celsius, and is not eating or drinking well. On examination, he has a temperature of 38.5 degrees Celsius, HR of 150, RR of 30. He has a blanching macular rash on his torso, swollen hands and feet, red eyes, red cracked lips, large tonsils with no pus, and a left-sided 2cmx3cm cervical lymph node which is mobile. There is no photophobia or neck stiffness. His chest is clear with normal heart sounds and his abdomen is soft with a palpable liver edge. You note his BCG scar is inflamed. What is the most likely diagnosis?

A) Viral tonsillitis
B) Bacterial tonsillitis
C) Meningitis
D) Hand, foot and mouth disease
E) Kawasaki's disease
A

E

The diagnostic criteria for Kawasaki disease is a fever for 5d or more, plus 4 of:

Conjunctivitis (bilateral, non-purulent)
Rash
Adenopathy (cervical)
Strawberry tongue (or cracked, red lips)
Hands (swollen, and feet)

A good mnemonic is CRASH and burn (for the fever).

BCG scar inflammation is common in Kawasaki but not necessary.

A viral tonsillitis would usually settle after 5d of fever, and does not explain the other symptoms.

Bacterial tonsillitis usually shows some pus.

Meningitis is unlikely in the absence of meningism.

Hand, foot, and mouth disease is a mild coryzal illness with mouth ulcers and painful papules on the hands and feet of young children. It’s caused by a coxsackie virus infection.

94
Q

A 14-year-old refugee from Afghanistan who has lived in the UK for 2 years comes to see you complaining of increasing fatigue and breathlessness on exertion. On examination, she appears cyanosed, and has bilateral basal fine crepitations and a soft pan systolic murmur with a displaced apex beat. She has never been in a hospital and has no surgical scars. You urgently refer her for a cardiology review. What is the most likely diagnosis?

A) Bacterial endocarditis
B) Tetralogy of Fallot
C) VSD producing a left-to-right shunt
D) Eisenmenger's syndrome
E) Ebstein's anomaly
A

D

Eisenmenger’s syndrome is a result of untreated VSD.

A large VSD can lead to left ventricular hypertrophy and then to left sided cardiac failure. This causes pulmonary hypertension and increases the pressure in the right-side of the heart, resulting in a right-to-left shunt through the VSD.

There are no infective features to suggest bacterial endocarditis.

Untreated tetralogy of Fallot would not survive to teenage years.

Left-to-right shunts are not cyanotic.

Ebstein’s anomaly is a heart defect that would not survive without surgery. It’s a combination of an abnormal tricuspid valve, hypoplastic right ventricle, and pulmonary stenosis.

95
Q

The major criteria for rheumatic fever include all of the following features except:

A) A new murmur
B) Swollen right knee for the past 8 days
C) A geographic shaped rash with central pallor on the abdomen
D) Involuntary movements of the arms
E) Fever

A

E

Rheumatic fever is a post-infectious complication of group A streptococcal infection, typically arising after an episode of tonsillitis. It requires 2 major symptoms or 1 major symptom and 2 minor symptoms, with evidence of a streptococcal infection such as a throat swab or positive antistreptolysin titre.

Major:
Pancarditis (endocarditis --> murmur or valve dysfunction; myocarditis --> heart failure; pericarditis --> rub, effusion, or tamponade
Polyarthritis (more than 1 week total)
Sydenham's chorea (2-6m post-infection)
Erythema marginatum
Minor:
Fever
Arthralgia
FH or PMH of rheumatic fever
Raised inflammatory markers
ECG changes (prolonged PR interval)
96
Q

A 5-year-old child was admitted overnight awaiting surgical repair of a broken right ankle, and was noted to have a raised BP consistently above 130/90mmHg despite adequate analgesia. On examination, he has a plaster on his right foot, and appears comfortable at rest. On auscultation, there is a soft systolic murmur heard at the right upper sternal edge. His femoral pulse is difficult to find, but present bilaterally. When felt with the radial pulse, the impulse in the femoral pulse occurs slightly later. His abdomen is soft and there are no bruits heard. There BP done in the right arm is 136/92 mmHg, but the left arm gives a reading of 124/80. What is the most likely diagnosis?

A) Normal blood pressure in the left arm with a spurious result from the right
B) Coarctation of the aorta
C) Renal artery stenosis
D) Phaeochromocytoma
E) White coat syndrome
A

B

Radiofemoral delay, differing BPs in R and L arms, and systolic murmur in the aortic area suggest coarctation of the aorta. It’s likely a mild stenosis which is why it hasn’t been picked up yet.

Renal artery stenosis would usually give a bruit in the abdomen, but abdominal Doppler could rule it out.

Phaechromocytoma is a rare cause of HTN, unlikely here.

97
Q

Which of the following is not a feature of cardiac insufficiency?

A) Scattered wheeze on auscultation of the chest
B) Central cyanosis
C) Sacral oedema
D) Tachypnoea with the apex beat palpable in the 7th intercostal space just lateral to the mid-clavicular line
E) Hepatomegaly

A

B

All may be present in cardiovascular disease, but central cyanosis tends to be due to either lung pathology or a right-to-left shunt. Therefore, the cyanosis seen in cardiac insufficiency is a result of lung pathology, not the cardiac insufficiency itself.

98
Q

40% of children of children with trisomy 21 have congenital heart defects. Which of the following is not associated with Down syndrome?

A) Tetralogy of Fallot
B) Atrioseptal defect (ASD)
C) VSD
D) Atrioventricular septal defect (AVSD)
E) Transposition of the great arteries
A

E

40% of Down cardiac anomalies are AVSDs. 30% are VSDs, 10% ASDs, and 6% Tetralogy of Fallot (including RVOT, VSD, overriding aorta, and RVH).

99
Q

A 14-year-old girl was seen in accident and emergency following her third collapse this year and referred to cardiology for review of a low rumbling murmur heard at the left upper sternal edge. Her ECG in accident and emergency was normal. Her blood sugar was 5.3 mol/L. Urea and electrolytes were normal. The most recent collapse occurred at school while waiting for exam results to be given out. Previously they occurred while watching a parade all afternoon standing in a crowded street, and at a party. On all three occasions, she felt dizzy beforehand, was unconscious for less than 10 seconds, and fully alert following the episode, but did feel nauseous. Her ECG today is normal. What is the most likely diagnosis?

A) Venous hum murmur and vasovagal syncope
B) Innocent murmur and epilepsy
C) Wolff-Parkinson-White syndrome
D) Patent foramen ovale and sick sinus syndrome
E) Neurocardiogenic syndrome

A

A

A low rumbling murmur heard above the nipple line with a normal ECG is a venous hum and normal in children. As her cardiac investigations are normal, this is likely vasovagal syncope.

There is no suggestion of convulsions or post-ictal state so it’s unlikely epilepsy.

Wolff-Parkinson-White syndrome shows delta waves on ECG.

Patent foramen ovale would be seen on echocardiograms, and sick sinus syndrome would give bradycardia on ECG.

Neurocardiogenic syndrome is diagnosed on a tilt table test, and would not normally have prodromal symptoms as here.

100
Q

A 1-month-old baby attends accident and emergency with a 2-day history of fever to 38.8 degrees Celsius measured at the GP surgery. He has been vomiting, with no diarrhoea, rash, cough, or coryza. A clean catch urine has leukocytes +++ and ketones, no nitrites, blood, or protein. An urgent microscopy shows >200 cells/uL white cells. What is the most appropriate course of action?

A) Discharge home with 3 days of trimethoprim
B) Admit for a course of IV antibiotics to cover a UTI
C) Admit for a lumbar puncture, blood cultures, and CXR, IV antibiotics
D) Organise an urgent DMSA scan
E) Discharge home with reassurance and advice to return if fever persists

A

C

This is a non-specific presentation. In children under 3m, a febrile child must be thoroughly investigated for a cause. He needs a septic screen (blood culture, urine culture, CSF culture, CXR, CRP)

Urine leukocytes are not sensitive on their own or in infants. If the urine culture grows >105CFUs/ml, then the child can be treated for UTI as in B. They will also require a DMSA scan 6 weeks post-infection (not urgently as acute settings can give misleading results), and micturating cystourethrogram depending on age.

101
Q

A 5-year-old boy presents to his GP with a 3-day history of puffy eyes. He has been unwell with a coryzal illness for the last week. His mother states he has had no new medications and no hayfever, allergies, or asthma. On further examination, he has generalised oedema and scrotal oedema. He is tachycardic and has cool peripheries, no skin rashes or erythema. What is the most likely diagnosis?

A) Periorbital cellulitis
B) Allergic reaction
C) Nephrotic syndrome
D) Nephrotic syndrome with hypovolaemia
E) C1 esterase deficiency
A

D

Nephrotic syndrome is a triad of proteinuria, hypoalbuminaemia, and generalised oedema. It can be precipitated by a intercurrent infection, as here. This child also has cool peripheries and tachycardia, so he has hypovolaemia due to third space losses due to reduced oncotic pressure.

Periorbital cellulitis does not explain his other symptoms, and usually presents with fever.

Allergic reactions would present with skin rashes, wheals, flares.

C1 esterase deficiency or hereditary angioedema presents around puberty with recurrent episodes of swelling in response to various triggers.

102
Q

A 12-year-old girl presents to her GP with a UTI. She has no past medical history of note, and is not taking any medication. On testing her routine observations, her BP was 140/90mmHg with a manual sphygmomanometer. You are concerned this may be high for her age. She has no headaches, visual disturbance, vomiting, chest pain, dyspnoea, or neurological signs. What is your next course of action?

A) Repeat the BP on three different occasions
B) Discuss the BP reading with a paediatric nephrologist
C) Commence sodium nitroprusside
D) Repeat the BP with an automated machine
E) Discharge home with reassurance

A

A

BP measurements can be altered or incorrect based on pain, anxiety, incorrect cuff size and poor technique. Abnormal measurements should be repeated, and if consistently high, paediatric nephrologists should be involved. Possible causes could include essential hypertension, renal (renal artery stenosis, CKD, Wilms’ tumour), cardiac (CoA), endocrine (Cushing’s, phaeochromocytoma, neuroblastoma), or metabolic (hyperaldosteronism, CAH).

Sodium nitroprusside is used in malignant hypertension, which is not this.

Manual BPs are better than automated machines.

103
Q

A 6-year-old girl presents to hospital with a large-right sided abdominal mass. It does not cross the midline. On further questioning, she has had macroscopic haematuria, and weight loss of 4kg over the last 4 months. She has reduced appetite and lethargy. Her BP is 125/73 mmHg, HR 120bpm. Which of the following is not a complication of this malignancy?

A) Malnutrition
B) Hypertension
C) Renal impairment
D) Urinary catecholamines
E) Metastatic spread
A

D

This is Wilms’ tumour. It can cause malnutrition by poor appetite, vomiting, and increased metabolic demand. It causes HTN by renal artery compression or renin production. It causes renal impairment if bilateral (5% of cases) or the other kidney is non-functional. Metastatic spread is usually to lungs or liver.

Wilms’ tumours do not cause sympathetic stimulation, but neuroblastoma does, so this is a potential differential.

104
Q

An 8-year-old boy recently attended accident and emergency with a swollen left ankle. He had an x-ray and was discharged him and told there was no fracture. He has now developed a dark purple rash on his legs, which does not disappear with a glass pressed on. He was brought back to the department today vomiting, with abdominal pain. His observations and urine dipstick are all within normal limits. What is the most likely diagnosis?

A) Diabetic ketoacidiosis
B) Viral gastroenteritis
C) Meningococcal sepsis
D) Idiopathic thrombocytopenia purpura
E) Henoch-Schonlein purpura
A

E

Henoch-Schonlein purpura is a (usually post-infectious) small vessel vasculitis causing symptoms of skin (purpuric rash on legs and buttocks), GI tract (abdo pain, gross or occult blood, intussusception), joints (swelling and pain of large joints), and kidneys (glomerulonephritis with microscopic haematuria).

105
Q

A 7-year-old boy presented to accident and emergency with diarrhoea and vomiting for the past week. He had no history of foreign travel, but had been to a zoo recently on a school trip. He was discharged home, after providing a stool sample, with rehydration advice as he was less than 5% dehydrated and tolerating oral fluids. The stool had grown Escherichia coli 0157 which was phoned from the microbiology laboratory to the on-call doctor 48 hours later. What is the most serious complication?

A) Acute kidney injury
B) Haemolytic uraemia syndrome
C) Severe hypernatraemic dehydration
D) Henoch-Schonlein purpura
E) Post-gastroenteritis syndrome
A

B

HUS is a result of Shiga toxin production by E. coli or Shigella. It causes renal vasculature thrombosis and infarction, leading to AKI, microangiopathic haemolytic anaemia and thrombocytopenia.

106
Q

A 5-year-old girl was brought to hospital at midnight by her mother with 5% partial thickness burns to her chest and abdomen. Her mother states that she pulled on the kettle at 2pm and the boiling water scalded her. On examination, she is tachycardic, and drowsy with cool peripheries. Her initial blood tests: Na 150mmol/L, K 7.8mmol/L, urea 10.2mmol/L, creatinine 104umol/L, Hb 14g/dL. What is the most likely aetiological factor to account for these results?

A) Post-renal cause of acute kidney injury
B) Poisoning
C) Renal cause of acute kidney injury
D) Dehydration
E) Pre-renal cause of acute kidney injury

A

E

This girl has presented with a scald injury. The accident happened 10 hours before presentation which may arouse suspicion of non-accidental injury, but at present the more concerning feature is the acute kidney injury in a child with clinical shock. Burns damage the protective skin barrier, which allows excessive fluid and electrolyte loss. She may have been losing fluid through the damaged skin during the day, causing worsening dehydration; however she is now so fluid depleted that she has pre-renal acute kidney injury and hypovolaemia.

107
Q

A 10-day-old baby boy was brought to accident and emergency with a distended abdomen. On questioning, he was born at term with no antenatal concerns. Until 2 days ago, he had been feeding well and not vomiting, he had been wetting nappies, but mother has not witnessed a good urinary stream. On examining the child, you find a mass, dull to percussion, arising out of the pelvis, and he has had no wet nappies for the last day. You suspect he may have posterior urethral valves. Which one test will help to diagnose this underlying condition?

A) DMSA scan
B) Renal biopsy
C) CT abdomen
D) Micturating cystourethrogram
E) Renal ultrasound
A

D

A micturating cystourethrogram (MCUG) is indicated in recurrent UTIs, or to look for urinary tract abnormalities.

A renal biopsy would not help to diagnose posterior urethral valves.

A DMSA scan looks for fibrosis or abnormalities of the kidney.

A CT abdomen could show the enlarged bladder, but not the urethral valves.

108
Q

A 12-year-old boy who was born with multicystic dysplastic kidneys had a renal transplant when he was 7 years old due to chronic kidney disease stage V, after having peritoneal dialysis for 1 year. Which of the following would you not expect him to be taking?

A) Septrin
B) Tacrolimus
C) Diclofenac
D) Growth hormone
E) Erythropoetin
A

C

Renal transplant requires immunosuppression with medications such as tacrolimus, ciclosporin, and mycophenolate mofetil. This predisposes to infection, and so septrin (co-trimoxazole: sulfamethoxazole and trimethoprim) is used prophylactically. Children with chronic renal disease often have impaired growth which is responsive to growth hormone. Erythropoeitin is produced in the kidneys, so supplementation is beneficial.

Diclofenac should not be used in renal impairment as it and other NSAIDs are nephrotoxic.

109
Q

An 11-year-old girl presents to the out of hours GP while on holiday in England with abdominal pain. She tells you she has polycystic kidney disease, diagnosed in early life. She has bilateral palpable kidneys, and hepatosplenomegaly, with visible distended veins on the abdomen, and ascites. Abdominal ultrasound shows liver fibrosis. What is the inheritance of this condition?

A) Autosomal dominant
B) X-linked
C) Sporadic mutation
D) Autosomal recessive
E) Microdeletion
A

D

There are two types of PKD, ARPKD and ADPKD. The autosomal recessive form presents in early childhood with bilateral masses, pulmonary hypoplasia, congenital hepatic fibrosis, and pulmonary hypertension. The autosomal dominant disease presents in older children or adults with renal, liver, and cerebral vasculature affected.

110
Q

An 11-year-old girl was brought to accident and emergency in December with pain in her left leg. She is known to have sickle cell disease and her baseline haemoglobin is 7.0g/dL. She has been admitted in the past with painful leg and chest crises. She has a cough and coryza. Today her blood results show: Hb 6.8, WCC 12x10^9/L, platelets 209x10^9/L, CRP 20mg/L. What is not part of the appropriate initial management?

A) IV fluids
B) 15L oxygen through a non-rebreather mask
C) Exchange transfusion
D) IV antibiotics
E) Oramorph
A

C

A vaso-occlusive crisis is triggered by cold weather, infection, hypoxia, or dehydration. Therefore, analgesia, rewarming, Abx, oxygen, and IV fluids are indicated. A top-up transfusion is indicated in marked drops in haemoglobin.

Exchange transfusion is only indicated in sickle chest crisis, stroke, and priapism.

111
Q

A 4-year-old boy is brought into accident and emergency with a limp for 1 day. He was unhappy to weight bear on his right leg. He had been with his grandparents all day and his mother brought him to hospital when she returned from work that evening. He was afebrile with a HR of 110bpm, but had a cold last week. Mum reports no history of trauma. What is the most important diagnosis to exclude?

A) Behavioural
B) Acute leukaemia
C) Reactive arthritis
D) Soft tissue injury
E) Septic arthritis
A

B

Acute leukaemia can present with limp due to bone pain secondary to dissemination of the disease. It is important to do a full blood count and blood film on these patients to look for pancytopenia and presence of blast cells. Though all options could be the cause, this is the most important diagnosis to exclude.

112
Q

A 14-year-old girl was diagnosed with idiopathic thrombocytopenia purpura (ITP) last week after she attended the children’s assessment unit with recurrent epistaxis. She had a platelet count of 16x10^9/L last week and now presents to accident and emergency with further episodes of epistaxis, haematemesis, and petechiae. She had a heart rate of 110bpm and her BP is 100/70mmHg. What is the next best management step?

A) Give a platelet transfusion and red cell transfusion
B) Arrange an urgent upper gastrointestinal endoscopy
C) Give IV immunoglobulin and steroids
D) Admit and monitor the haemodynamic status and administer a fluid bolus
E) Discharge home with advice to return if the symptoms continue for more than 24 hours

A

D

This girl is in hypovolaemic shock. The correct course is to admit, monitor her haemodynamic status, and give a fluid bolus.

ITP is self-limiting, and treatment can be supportive as an outpatient. However, they may require more intensive treatment.

Platelet transfusions are only given in life-threatening bleeding, as new platelets will be destroyed by the same immune-mediated mechanism.

Endoscopy may cause more harm than good.

IVIG and steroids can be used to reduce underlying destruction, but bone marrow aspirate should be carried out first to exclude malignancy.

113
Q

A 14-year-old girl went to her GP with a sore throat and cervical lymphadenopathy. She had a blood test done and you are called later that day with results: haemoglobin 6.0g/dL, WCC 230x10^9/L, neutrophils 0.9x10^9/L. Blood film showed blasts and Auer rods. What is the most important management priority for this child in the first 24 hours from diagnosis?

A) Overwhelming sepsis
B) Febrile neutropenia
C) Heart failure
D) Uncontrollable bleeding
E) Tumour lysis syndrome
A

E

Auer rods give the diagnosis: AML. Her WCC is very high, causing pancytopenia of all other cell lines. Although all are potential complications of AML, the first management step is to prevent tumour lysis syndrome.

As cells break down in treatment, hyperkalaemia, hyperphosphataemia, gout, fluid overload or dehydration, raised urea, and raised creatinine can occur. Prophylactic hyperhydration, allopurinol or rasburicase, and monitoring of electrolytes is indicated.

114
Q

A 2-year-old boy is admitted to the paediatric ward with a swollen, painful left knee. He has been afebrile and has a history of minor trauma to his knee earlier today. His mother is a haemophilia carrier and his father is not affected. You are keen to rule out haemophilia in this child. Which two clotting factors should you test for?

A) Factor VII and IX
B) Factor VII and VIII
C) Factor V and VI
D) Factor VIII and IX
E) Factor X and XI
A

D

Haemophilia A is due to a VIII deficiency, and B to IX. Severity of disease is based on the proportion of functional factor (mild 10-50%, moderate 2-10%, and severe <2%).

Treatment is with replacement of factors.

115
Q

A 20-year-old man presents to the infectious disease department with a large 7cmx8cm swollen painful lump in the left anterior triangle of his neck. He has night sweats, 10kg weight loss, and a dry cough for the last month. He was treated with surgery and radiotherapy for a high grade astrocytoma when he was 8 years old. Which of the following is not a recognised complication of his childhood condition and its treatment?

A) Finger clubbing
B) Haematological malignancy
C) Educational difficulties
D) Short stature
E) Infertility
A

A

Finger clubbing has respiratory causes (malignancy, fibrosis, suppurative disease), cardiac causes (cyanotic heart disease, infectious endocarditis, atrial myxoma), and gastrointestinal causes (malignancy, IBD, lymphoma, cirrhosis, coeliac disease).

Childhood malignancies treated with radiotherapy can have complications with secondary malignancies, usually lymphomas and leukaemias. Educational difficulties can follow, due to long periods of missesd school or poor ability to concentrate, see, or hear. Short stature can be a result of spinal and endocrine effects of radiotherapy. Irradiation and chemotherapy can both cause sterility,

116
Q

A 12-year-old girl has been seeing her GP for the last year with heavy periods and had suffered with bleeding gums when she was younger. She is otherwise well and lives with her adoptive parents who now have parental responsibility. Her coagulation tests reveal normal prothrombin time (PT) and activated partial thromboplastin time (APTT), low factor VIII, low von Willibrand factor (vWF), abnormal platelet aggregation, and increased bleeding time. What is the likely inheritance of her condition?

A) Autosomal dominant
B) Autosomal recessive
C) X-linked
D) Robertsonian translocation
E) Sporadic mutation
A

A

This is von Willibrand’s disease, likely type 1, given her mild symptoms. Type 1 and type 2 are autosomal dominant, and type 3 is autosomal recessive.

117
Q

A 4-year-old girl has just returned from holiday in France where she visited a petting farm. She has had diarrhoea for 2 days, and her mother noticed fresh red blood mixed with the stools. She has also been vomiting. On admission to hospital, her blood tests showed: Hb 5g/dL, WCC 15x10^9/L, platelets 55x10^9/L, urea 19mmol/L, creatinine 110umol/L. Her stool culture is pending. What is the most likely diagnosis?

A) Platelet disorder
B) Inflammatory bowel disease (IBD)
C) Severe dehydration
D) Henoch-Schonlein purpura (HSP)
E) Haemolytic uraemia syndrome (HUS)
A

E

Haemolytic uraemic syndrome is a microangiopathic haemolytic anaemia secondary to E. coli or Shigella gastroenteritis. Shigatoxin enters the blood and causes inflammation in the renal vasculature, which may lead to renal impairment. As with other MAHAs, Hb will be low, reticulocytes up.

Platelet disorders can lead to mucosal bleeding, however, the renal impairment is out of keeping.

IBD rarely presents in this age group.

Henoch-Schonlein purpura is a small vessel vasculitis that causes symptoms of the skin, GI tract, joints, and kidneys.

118
Q

An 18-month-old boy presented to the GP with a history of eating soil. He had been in the garden this afternoon as his mother put the washing out. She found him eating the soil and took him straight inside. On examination, he is well and alert, but has pale conjunctivae. He is not tachycardic or tachypnoeic. His diet consists predominantly of breast milk. What is the most likely result of his haemoglobin and haematinics?

A. Hb 10 g/dL, MCV 80 fl, ferritin normal, iron normal, vitamin B12 and folate normal
B. Hb 6.5 g/dL, MCV 100 fl, ferritin normal, iron normal, vitamin B12 and folate low
C. Hb 5.5 g/dL, MCV 55 fl, ferritin low, iron low, vitamin B12 and folate normal
D. Hb 7 g/dL, MCV 70 fl, ferritin normal, iron normal, vitamin B12 and
folate normal
E. Hb 6.8 g/dL, MCV 65 fl, ferritin normal, iron low, vitamin B12 and folate normal

A

C

This child is exhibiting pica, a phenomenon of eating substances that are not food, secondary to severe iron deficiency anaemia. This would lead to a microcytic anaemia, with low iron, and low iron stores (ferritin). This is likely due to prolonged breast-feeding and delayed weaning.

Children have sufficiency iron stores for 4m after birth, after which they require supplementation, usually by weaning onto pureed or solid foods.

119
Q

A 9-month-old boy presented to his GP with lethargy and a prominent forehead. He is pale on examination, and has yellow sclerae. He is the first child of his non-consanguineous parents. His haemoglobin is 6.5g/dL, WCC 5.0x10^9/L, platelets 300x10^9/L. His blood film shows evidence of haemolysis, no spherocytes, no sickle cells, and a good reticulocyte count. Direct anti globulin test (DAT) is negative. What is the most likely diagnosis?

A) Beta thalassaemia
B) Sickle cell disease
C) ABO incompatibility
D) Hereditary spherocytosis
E) G6PD deficiency
A

A

Beta thalassaemia is a haemolytic anaemia with autosomal recessive inheritance. It generally presents after 6m of age, when HbA is produced. Symptoms include pallor, jaundice, poor feeding, decreased activity, and extramedullary haematopoeisis (skull bossing).

120
Q

A 3-year-old is brought into accident and emergency on a Monday morning because she has developed several bruises on her buttocks, left leg, and right arm. She is seen with her nanny, who reports finding the bruises when she was getting her dressed this morning. Recently, the girl has not been herself. She has had several colds over the past 2 months, and has been more lethargic lately. The nanny is worried she is losing weight. On examination, she appears withdrawn, pale, and has a bruise on the left buttock which is 5cmx8cm. She has three other bruises on her left leg and right arm which are of varying colours. She also has some fine petechiae on her neck and cheeks. She has a runny nose and a cough, but the chest is clear. What is the most likely diagnosis?

A) Non-accidental injury
B) Leukaemia
C) Idiopathic thrombocytopenia
D) Henoch-Schonlein purpura
E) Accidental injury
A

B

This child is presenting with bruising and petechiae which suggests low platelets. This directs away from NAI, accidental injury, and HSP. The background of being unwell and losing weight suggests leukaemia over ITP.

121
Q

On the day 1 baby check, a mother is very concerned about a rash on her baby’s face. Over the right eye, forehead, and temple, there is a pink-red, flat area of erythema. He is opening the eye, and his eye movements seem intact. The child’s observations and rest of the examination are normal. What should you tell the mother?

A) This is a strawberry naevus, and it may get bigger before it goes away by about 5 years of age
B) This is a port wine stain and the baby needs an MRI scan to check for intracranial involvement
C) This is a capillary haemangioma (stork mark) and is normal; it will fade over the first year or so of life
D) This is orbital cellulitis and he needs IV antibiotics
E) This is erythema toxicum, which is a normal baby rash and will go away within the first few weeks

A

B

This is a port wine stain. They are flat erythematous areas that remain for life. They’re generally benign, but when in the trigeminal area they can involve the central nervous system in Sturge-Weber syndrome, so a head MRI is indicated.

Strawberry naevi appear in the first few weeks as flat patches that grow over the first few years, and then fade away.

Stork marks are present from birth as flat erythematous areas in the midline of the head, usually the forehead, eyelids, or nape of the neck. They disappear within the first year.

Cellulitis is unlikely due to the normal observations.

Erythema toxicum is an erythematous rash with pustules that comes and goes over the body during the first few weeks of life.

122
Q

A mother brings her 6-month-old, formula-fed baby to see the GP complaining that the olive oil she is using is not helping his persistent cradle cap, and worsening rash on his face and arms. On examination, he has extensive cradle cap and eczematous changes on his cheeks, neck, chest, and arms. The neck skin creases are red and oozing with yellow crusts. He is miserable, and feels warm to touch. What is the most appropriate management?

A) Advise using emollients and a soap substitute
B) Start emollients with a topical antibiotic
C) Refer to hospital for intravenous antibiotics
D) Recommend a trial of switching to soya based formula, as he may be cow’s milk protein allergic
E) Start topical steroids on the inflamed areas, and intensive emollient treatment

A

C

This is infected eczema. The yellow crusts suggest staphylococcal infection, but eczema herpeticum needs to be ruled out. He’s systemically unwell, so needs IV antibiotics.

If he was not unwell, emollients and oral antibiotics could be appropriate, but not topical, as they can be sensitising.

Once the infection is resolved, he can be sent home with emollients, soap substitute, and topical steroids.

123
Q

A 4-year-old is brought into accident and emergency by very anxious parents. She has had a bad cough, which makes her vomit, and a fever for 2 days. She has now developed a rash on her face which does not pass the glass test, in that the spots are still visible when a glass is pressed against the skin. On examination, she is alert and comfortable at rest, with fine petechiae on her cheeks and neck, which are non-blanching. She has red, enlarged tonsils, without pus, and the chest is clear. What is the most likely cause of her rash?

A) Meningococcal sepsis
B) Idiopathic thrombocytopenia
C) Henoch-Schonlein purpura
D) Non-accidental injury
E) Capillary rupture secondary to raised pressure in the superior vena cava distribution
A

E

Her petechiae are in the SVC distribution, and this can occur in coughing or vomiting, but also in shaken babies and strangulation injuries. This is most likely due to her being systemically well, but meningococcal sepsis is a possibility, so she should be admitted and observed anyway.

ITP occurs about 1 week after a throat infection with petechiae, easy bruising, and bleeding, but otherwise well, with no remaining signs of the preceeding throat infection.

HSP is a small vessel vasculitis causing symptoms of the skin, GI tract, kidneys, and joints. The purpuric rash is usually over the buttocks and legs, so not as in this vignette.

124
Q

A 2-year-old was seen in accident and emergency by the senior house officer with a short history of fever, malaise, and now vomiting. She had a blanching rash on her arms and abdomen. She looked unwell, but had no clear focus for her fever. She was tachypnoeic, but her chest was clear. A urine sample was requested which showed a trace of leukocytes and ++ ketones. 45 minutes later, the paediatric registrar came to review the child who appears lethargic with a capillary refill centrally of 6 seconds. The rash on her abdomen is now non-blanching. What is the most likely diagnosis?

A) Urinary tract infection (UTI)
B) Idiopathic thrombocytopenia
C) Meningococcal sepsis
D) Human herpes virus 6 infection
E) Diabetic ketoacidosis
A

C

This child has a non-blanching rash, shock, and fever. This suggests she has DIC, as in meningococcal sepsis.

A UTI would most likely show high leukocytes, nitrates, and potentially blood in the urine dip, and does not explain her systemic disease.

ITP occurs about 1 week after a throat infection with petechiae, easy bruising, and bleeding.

Human herpes virus 6 causes roseola, which consists of 3-5d of fever followed by an asymptomatic erythematous maculopapular rash.

DKA doesn’t match this vignette, and glucose would have been seen in the urine dip.

125
Q

A 2-year-old is brought in by ambulance after pulling a pot of boiling water off the stove down on top of himself. He has significant burns to the whole of his face, torso, and right arm. Estimate the percentage body surface area affected.

A) 20%
B) 30%
C) 40%
D) 50%
E) 60%
A

C

The rule of 9s is useful for estimating percentage SA in over 15yo (9% for head, chest, abdomen, upper back, lower back, left arm, right arm, left thigh, right thigh, left calf, right calf). Under 15, the head accounts for a much higher percentage, with 20% of surface area being the head in under 1y.

Therefore, using the rule of 9s, he has 31.5%, but taking into account his age, this is closer to 40%.

126
Q

A 3-year-old is brought to see the GP with multiple pearly raised papule with central umbilications. They have been there for more than a month on his torso and upper legs. His mother is worried he has warts. What is the most likely diagnosis?

A) Molluscum contagiosum
B) Congenital warts
C) Scabies
D) Melanocytic naevi
E) Guttate psoriasis
A

A

This child has a common condition of molluscum contagiosum caused by a pox virus. They are usually self-limiting and will self-resolve within a year.

127
Q

On a newborn baby check of an Asian, 36-hour-old baby, you note a large bruise coloured area on the buttocks and lower back which seems non-tender. The mother does not know how it got there. He is handling well, and the rest of the baby check is unremarkable. What is the most likely explanation?

A) Non-accidental injury
B) Mongolian blue spot
C) Neonatal sepsis with disseminated intravascular coagulation
D) Idiopathic thrombocytopenia purpura
E) von Willebrand's disease
A

B

128
Q

A 5-day-old baby is brought to see the GP because she has had a rash for the past 3 days which started on her chest, is spreading to her face, and getting worse. On examination, she handles well, and is alert. There is an erythematous rash on her face, torso, and right arm, with little pustules. What is the most likely diagnosis?

A) Infected eczema
B) Neonatal sepsis
C) Neonatal acne
D) Molluscum contagiosum
E) Erythema toxicum
A

E

129
Q

A 10-year-old girl was diagnosed with diabetes 1 year ago. She has been compliant with her insulin regimen and her HBA1C is 6%. She is attending her annual diabetic review, and has been asking about why she has diabetes. What is the aetiology of type 1 diabetes?

A) Inflammation of the pancreas causing exocrine and endocrine dysfunction
B) Impaired glucose tolerance
C) Secretory dysfunction of the pancreatic duct
D) Autoimmune destruction of pancreatic islet cells
E) Peripheral insulin resistance

A

D

130
Q

A 2-year-old girl was brought by her mother to accident and emergency after 4 days of vomiting and abdominal pain. She had brought her in 2 days ago after developing a cold and was discharged home and diagnosed with a ‘tummy bug’. On examination, she was drowsy, had dry mucous membranes, deep heavy breathing, cool peripheries, and tachycardia. Her mother reports a 1-month history of weight loss, excessive drinking, and passing large volumes of urine prior to this episode. Her urine dipstick has ketones and glucose. Her blood gas shows the following: pH 7.10, PCO2 2.1kPa, PO2 10kPa, BE -12, HCO3 18mmol/L. What is the most likely diagnosis?

A) Severe dehydration secondary to gastroenteritis
B) Sepsis secondary to gastroenteritis
C) Diabetic ketoacidosis (DKA)
D) Chronic kidney disease
E) Hyperosmolar hyperglycaemic non-ketotic state

A

C

131
Q

A 3-week-old baby is brought to the ‘prolonged jaundice clinic’. His mother reports he has poor feeding, is not gaining weight appropriately and is more sleepy compared to her previous child. He opens his bowel once a day and is being mix breast and bottle fed. He is floppy, jaundiced, has a large, protruding tongue, and a hoarse cry. He had a newborn blood spot screening test done at birth which was normal and he has no dysmorphic features. What is the most likely diagnosis of this child?

A) Beckwith-Wiedemann syndrome
B) Congenital hypothyroidism
C) Down's syndrome
D) Normal baby
E) Prader-Willi syndrome
A

A

This child has many features of hypothyroidism. Children with prolonged jaundice should have their thyroid function tested. The newborn blood spot tests for high thyroid-stimulating hormone (TSH) levels in response to low thyroxine, which should detect most congenital hypothyroidism. Rarely, in children with
panhypopituitarism, there is low TSH and therefore low thyroxine.

Beckwith-Wiedemann syndrome causes foetal macrosomia, hyperinsulinaemia leading to hypoglycaemia, hemihypertrophy, and abnormal ear development. This would not explain the jaundice.

Prader-Willi syndrome causes hypotonia and poor feeding, but not the macroglossia or jaundice.

132
Q

A 10-year-old girl with Graves’ disease attends her GP with worsening of her symptoms. She was well controlled on carbimazole and has had relatively few symptoms for the past 6 months. She now has sweats, weight loss, diarrhoea, and tremors, which are affecting her school performance. What is the next management step?

A) Review in 3 months
B) Radioisotope therapy
C) Stop carbimazole and start propranolol
D) Optimise carbimazole dose and add propanolol
E) Referral for thyroidectomy

A

D

133
Q

A 4-year-old boy was diagnosed with nephrotic syndrome 6 months ago and has required a long coarse of oral corticosteroids to maintain remission of the condition. He has developed truncal obesity and you are concerned he may be developing Cushing’s syndrome. Which of the following is not a complication of Cushing’s syndrome?

A) Osteoporosis
B) Short stature
C) Gastric irritation
D) Hypertension
E) Hypoglycaemia
A

E

134
Q

A 13-year-old girl has presented to her GP with her mother with concerns that she is the shortest in her class at school. She has always been ‘on the small side’ according to her mother, despite eating well. When you examine her you find she is normal. She has no signs of pubertal development and you notice she has widely spaced nipples and a low hair line. You are considering the diagnosis of Turner’s syndrome What is the most appropriate diagnostic investigation?

A) Mid parental height
B) Echocardiogram
C) Four limb blood pressures
D) Karyotype
E) Fluorescence in situ hybridisation (FISH)
A

D

135
Q

You are asked to examine a tall 15-year-old boy. His height is above the 98th centile for his age and he has other concerns about the development of breast tissue. He was told this was normal as he develops through puberty, but his father states he has no facial or underarm hair. He allows a brief examination of his genitalia and you note he has a small penis and testicular volume. He has no arachnodactyly or visual problems. What is the most likely diagnosis?

A) Delayed onset of puberty
B) Klinefelter's syndrome
C) Precocious puberty
D) Marfan's syndrome
E) Normal variation
A

B

136
Q

A 16-year-old boy attends your GP clinic for the first time with his father. He has recently moved to the area. His father is concerned that he is shorter than his peers at school, and he frequently complains about being bullied. On further questioning, there is no evidence of chronic or familial illness, and he eats a balanced diet. His weight is on the 25th centile, and his height is on the 10th centile. On examination, he has no evidence of facial, axillary, or pubic hair. His testes are both descended and are <4mL volume. What is the most likely cause of his delayed puberty?

A) Anorexia nervosa
B) Hypothalamo-pituitary dysfunction
C) Kallmann's syndrome
D) Cryptorchidism
E) Constitutional delay
A

C

137
Q

A 6-year-old girl has presented to her GP with a rapid increase in growth. Her mother is also concerned that she seems to have developed pubic and axillary hair and breast development prior to this, but thought it would go away. She has no history of trauma, but has reported problems with her vision. Her levels of GnRH, FSH, LH, and oestrogen are high. You are concerned that she may have a pituitary tumour. What is the likely visual field defect?

A) Monocular blindness
B) Central scotoma
C) Homonymous hemianopia
D) Bitemporal hemianopia
E) Myopia
A

D

138
Q

A 9-year-old girl presents to accident and emergency with fever, vomiting, and dysuria. She is wearing a steroid bracelet, and has a steroid card stating she is on daily prednisolone for severe asthma and eczema, and is therefore at risk of adrenal suppression. She is tachycardic at 140bpm, and you are concerned that her BP is low. Her capillary glucose is 3.0mmol/L. What is the single most important investigation?

A) Cortisol
B) FBC
C) Renal function 
D) Urine culture
E) Blood culture
A

C

139
Q

A 3-year-old is brought to accident and emergency by his parents because he has not been walking for the past day and refuses to stand. He is normally fit and healthy, but he did have antibiotics for tonsillitis 2 weeks ago. They do not think he has had any injuries, but he attends daycare, and something could have happened there. He is up to date with his immunisations, and his parents have no concerns with his development. On examination, he looks well, is apyrexial, has a HR of 120bpm, and a RR of 26, with no bruising. His knees are normal on examination, and the hips have a full range of movement, except he cries on external rotation of the right hip. There are no deformities seen on x-ray of the hips and knees. After some paracetamol, he manages to stand and take a few antalgic steps with encouragement, limping on the right leg. What is the most likely diagnosis?

A) Reactive arthritis
B) Non-accidental injury
C) Growing pains
D) Osteomyelitis
E) Septic arthritis
A

A

Section 22

140
Q

A 6-year-old girl is taken to see her GP because she is complaining of knee and elbow pains frequently. Her mother thinks it is worst after her ballet classes, and when she gets home from school. She denies stiffness or pain in the mornings. Her mother has been administering paracetamol several times a week, and is worried that this is too much to be giving a child. On examination, the child looks well and has full range of movement of her joints, with evidence of hyperextension. There are no swollen joints or effusions present, and she is non-tender on examination. What is the most likely diagnosis?

A) Repetitive strain injury
B) Marfan's syndrome
C) Hypermobile joints
D) Osteoarthritis
E) Juvenile idiopathic arthritis (JIA)
A

C

141
Q

A 14-year-old, slightly overweight boy is brought into accident and emergency from a football match where he slipped and fell, but was unable to get back up due to pain in his right leg, which is now looking shortened and externally rotated. X-rays show the right femur to be disconnected from the femoral head almost completely at the level of the epiphysis. What is the most appropriate management?

A) Analgesia, nil by mouth (NBM) until emergency internal fixation can be performed
B) Antibiotics and NBM while waiting for an open reduction operation
C) Analgesia and bed rest with traction until healed
D) Analgesia and a hip spica cast
E) Reassure and mobilise with physiotherapy as tolerated

A

A

142
Q

What is a greenstick fracture?

A) The classic pattern of vertebral column fractures associated with abuse by being hit with a cane or ‘greenstick’
B) A fracture of the distal radius and ulna with dorsal displacement associated with a fall on the outstretched hand
C) A fracture of the distal radius and ulna with ventral displacement
D) A fracture of the long bones in young children where only one cortex is broken, and the other is buckled
E) A fracture of the long bones in young children where the cortex is buckled on one side of the bone with no cortex separation on the opposite side

A

E

143
Q

A 2-year-old boy is brought to accident and emergency for the sixth time, and is found to have a right-sided, non-displaced, transverse fracture of his tibia. His parents state that he was running in the living room and tripped, landing on a toy truck. He has broken his other leg twice, several fingers, and his right arm previously. His mother is very upset. She is 5 months pregnant with their second child, and her anomaly scan yesterday suggested the baby has a broken leg. What is the most likely explanation for these fractures?

A) Osteogenesis imperfecta
B) Domestic violence and child abuse
C) Osteopetrosis
D) Achondroplasia
E) Clumsy child
A

A

144
Q

A 2-month-old baby is brought in by the babysitter because he has been crying since she arrived to look after him, and his right leg looks swollen. He is the only child in the household. She does not think he is moving it, and is worried it is injured. On examination, he is miserable, his HR is 160bpm, RR 56, and capillary refill less than 2 seconds. He has a swollen right thigh. He cries more when that leg is being examined. You note a yellow bruise on his left thigh, and two purple bruises on either arm. X-rays show a fracture of the right femur, but the arms appear intact. A chest x-ray shows three healing posterior rib fractures. You are highly suspicious of non-accidental injury. What is the most appropriate management?

A) Give analgesia and plaster the leg fracture. Ask the babysitter to bring him back with the parents because he needs to be admitted
B) Give analgesia. Call the duty social worker on-call to get permission to discharge him once his leg has been plastered
C) Give analgesia and plaster the leg fracture. Contact the parents and inform them that he needs to be admitted. Ask them to come to the hospital and inform social services once they have arrived and been updated
D) Give analgesia and plaster the leg fracture. Call the police to bring the parents to hospital
E) Give analgesia and plaster the leg fracture. Call the police to arrest the babysitter for child abuse

A

C

145
Q

On a newborn baby screening examination, you see a baby girl born by elective C section for breech presentation. This is her mother’s first child. The examination is normal, except for a clunk felt on Barlow’s test, and a relocation click on Ortolani’s manoeuvre on the right side. What is the next step in management?

A) Refer to orthopaedics
B) Arrange an ultrasound for 6 weeks of age
C) Refer to physiotherapy
D) Ask a midwife to put on a plaster hip spica
E) Explain to the parents a watch and wait management is most appropriate as most self-resolve

A

D

146
Q

Which of the following is not a correct match?

A) Systemic JIA - acute illness with daily fevers, malaise, failure to thrive, rash, muscle and joint aches for greater than 6 weeks associated with raised inflammatory markers
B) Extended oligoarthritis - an arthritis originally affecting one or two joints for the first 6 weeks and over time has spread to multiple joints
C) Psoriatic arthritis - presents with interphalangeal joint swelling, scaly skin rash, nail pitting, and dactylics
D) Polyarticular arthritis - more common in boys, affecting multiple small joints for more than 6 weeks
E) Enthesitis-related arthritis - associated with HLA-B27 tissue type, and presents in older boys with large joint arthritis, swollen tender tendons, sacro-ilitis, and bamboo spine on x-ray. It is associated with anterior uveitis which, if left untreated, may cause blindness

A

E

147
Q

A 4-year-old is brought to accident and emergency acutely unwell and refusing to walk for the past 2 days. Her parents are not aware of any recent injuries. On examination, she is pyrexial, capillary refill 3 seconds centrally, HR 150bpm, RR 44/min. Her right thigh is swollen and slightly erythematous, but too tender to examine fully. An x-ray of the hip and femur shows soft tissue swelling surrounding the proximal femur, but the bones look normal. An urgent MRI shows a periosteal reaction in the proximal femur with extensive inflammation in the surrounding soft tissues. What is the most likely diagnosis?

A) Osteomyelitis
B) Non-accidental injury
C) Cellulitis
D) Reactive arthritis
E) Juvenile idiopathic arthritis
A

A

148
Q

A 5-year-old is referred to paediatrics due to concerns initially raised by his school teacher that he is weak and clumsy. On examination, he has wasting of his quadriceps and walks in a waddling gait. His blood creatinine kinase is 1600mmol/L (normal is 24-190). What is the most likely diagnosis?

A) Muscular dystrophy
B) Neglect with failure to thrive
C) Malnutrition with failure to thrive
C) Acute myositis
E) Spinal muscular atrophy
A

A

149
Q

A 3-year-old is brought into accident and emergency by ambulance following a generalised tonic clonic seizure that lasted 2 minutes. She did not require any treatment to stop the seizure, but on arrival the ambulance crew measured her temperature as 39.2 degrees Celsius and gave paracetamol. She is now pyrexial with a HR of 140bpm, RR of 30/min, and capillary refill less than 2 seconds. On examination, she has red enlarged tonsils with no pus, no neck stiffness, or rash. What is the most appropriate management?

A) Oral penicillin
B) Lumbar puncture and IV ceftriaxone
C) Explain that this was a febrile convulsion and discharge home
D) Start phenytoin
E) Discharge home with rescue buccal midazlolam for future seizures

A

C

Section 23

150
Q

A 7-year-old is referred to neurology due to frequent episodes of day-dreaming at school where she is unresponsive. She is falling behind in her work because of this. An electroencephalograph (EEG) shows three spike waves per second activity in all leads. What is the most likely diagnosis?

A) Temporal lobe epilepsy
B) Absence epilepsy
C) Day-dreaming
D) Benign Rolandic epilepsy
E) Narcolepsy
A

B

151
Q

A mother brings her 2-year-old daughter to the GP on a Monday morning. Over the weekend, she became very upset on being told ‘no’. She was screaming and then held her breath, went blue and fainted. She woke up quickly and seemed okay afterwards. However, it has just happened again this morning when she found some scissors and her mother took them away. On this occasion, she had a brief generalised convulsion lasting about 10 seconds. What is the most likely explanation?

A) Breath holding attacks
B) Reflex anoxic seizures
C) Absence epilepsy
D) Wolff-Parkinson-White syndrome
E) Vasovagal syncope
A

A

152
Q

A mother with known placenta praaevia with heavy vaginal bleeding was rushed into the labour ward and delivered by emergency C section at 35 weeks’ gestation. Pre-delivery the foetus was bradycardic and after birth APGARs were three at 1 minute, five at 5 minutes, and nine at 10 minutes. 36h later, on the special care baby unit, the baby is irritable and requiring NG tube feeds, as he is not sucking well .The tone in his upper limbs is reduced, and an EEG showed seizure activity which has been controlled by IV phenobarbitone. His cranial ultrasound is normal. His blood sugar monitoring is between 3.5 and 5mmol/L, CRP was less than 5mg/L, and is 7mg/L today. He is pyrexial. What is the most likely diagnosis?

A) Intraventricular haemorrhage
B) Group B streptococcal meningitis
C) Hypoglycaemia
D) Mild hypoxic ischaemic encephalopathy (HIE)
E) Moderate HIE
A

E

153
Q

A 15-year-old girl comes to accident and emergency complaining of sudden right arm weakness and double vision. Last week, she was incontinent of urine twice. She is normally fit and well. On examination, she has a left-sided 6th nerve palsy, and 4/5 power in her right arm. The examination is otherwise unremarkable. An MRI head shows multiple hyperintense, inflammatory, white matter lesions. What is the most likely diagnosis?

A) Brain metastasis
B) Multiple sclerosis
C) Tuberous sclerosis
D) TB meningitis
E) Neurofibromatosis
A

B

154
Q

A 15-year-old girl is brought into accident and emergency from school having disclosed to a friend that she took 10 paracetamol tablets last night. Her blood level of paracetamol is below the treatment line, her liver function tests and clotting are normal. Her father died of a brain tumour 3 years ago and her mother is being treated for reactive depression. The girl tells you she has been feeling low lately, particularly because she does not think she will do well in her up-coming exams. She regrets taking the tablets and does not think she will do it again. What is the most appropriate management?

A) Admit for monitoring of liver function and Child and Adolescent Mental Health Services (CAMHS) assessment
B) Refer to CAMHS as an outpatient and discharge as not currently suicidal
C) Refer to Social Services
D) Start antidepressant - fluoxetine
E) Start IV Parvolex

A

B

155
Q

Which of the following is not a feature of UMN lesion?

A) Slow-relaxing Achilles tendon reflex
B) Brisk reflexes
C) Increased tone
D) Decreased power
E) Up-going plantar reflex
A

A

156
Q

Which of the following is not a feature of raised intracranial pressure (ICP)?

A) Headache
B) Morning vomiting
C) Sun setting eyes
D) Bulging anterior fontanelle
E) Papilloedema
A

B

157
Q

A 10-year-old girl with sickle cell disease presents to her GP on Monday morning complaining of weakness in her right leg. She says she collapsed on Saturday afternoon and has not felt right since. What is the most likely diagnosis?

A) Sickle cell painful crisis
B) Parvovirus B19 infection
C) Aplastic crisis
D) Cerebral infarction
E) Osteomyelitis of the right femur
A

D

158
Q

A 6-year-old boy is registering with a new GP, having just moved to the area. He is in a wheelchair, but is able to mobilise with a fast, scissoring gait over short distances. He has increased tone in his legs and scars from previous tendon release surgeries. His upper limbs are normal. His mother says that his school performance is good, and he is writing well. She thinks he was going to have a Statement of Special Educational Needs assessment before they moved. As the GP, what is the most appropriate next step in management?

A) Reassure this mother that as he is doing well at school he does not need a statement
B) Refer to a community paediatrician
C) Refer to the physiotherapists and occupational therapists
D) Liaise with this new school teacher to make sure the school is able to support his physical needs
E) Refer to an educational psychologist

A

D

159
Q

A 6-year-old boy is taken to see the GP by his mother because he has been getting severe abdominal pains, sometimes with vomiting, and yesterday with a headache as well. He has no diarrhoea or constipation. His growth and examination are normal. He has no significant past medical history. In his family history, his maternal grandfather recently died of gastric cancer and mum’s migraines have been worse since his death. She is worried her son is getting gastric cancer too. What is the most likely diagnosis?

A) Crohn's disease
B) Brain tumour
C) Somatisation disorder
D) Gastric cancer
E) Coeliac disease
A

C

160
Q

A 13-year-old Somali girl presents to accident and emergency with a 1-month history of headaches, weight loss, and night sweats. Her father is concerned that she seems confused and is more unwell with her headache despite paracetamol. She was born in the UK and has had all her immunisations. She travelled to Somalia 6 months ago. The rest of the family is well although dad has a cough. On examination, she is thin and looks unwell, but is neurologically intact with no abnormal findings on clinical examination. Which diagnosis needs to be ruled out first?

A) Brain tumour
B) TB meningitis
C) Pulmonary TB
D) Migraines
E) HIV infection
A

B

161
Q

A 4-month-old baby being investigated for infantile spasms is noted to have an ash leaf macule on his back under Wood’s light. His EEG shows hypsarrythmia. The report of his MRI brain states there are subependymal nodules. What is the diagnosis?

A) Neurofibromatosis type I
B) Neurofibromatosis type II
C) West's syndrome
D) Tuberous sclerosis
E) Tay-Sachs disease
A

E

162
Q

A 3-month-old is brought into accident and emergency with a generalised tonic clonic seizure. She is pyrexial and the seizure stopped after 15 minutes with rectal diazepam given by the ambulance crew. Her HR is 130bpm, RR 36/min, and capillary refill less than 2 seconds. On examination, she is drowsy, has a port wine stain on her forehead, but is otherwise normal on examination. What is the most likely cause of her seizure?

A) Sturge-Weber syndrome
B) Tuberous sclerosis
C) Neurofibromatosis type I
D) Meningitis
E) Neurofibromatosis type II
A

B

163
Q

A 20-month-old boy has been referred due to delayed walking. On further questioning, you establish he has no difficulty feeding, had head control at 3 months of age, and sat up by 8 months. He has been crawling for the last 8 months, but he does not pull to stand or walk with support. He has no dysmorphic features. There is no known family history of muscle problems. His mother has no myotonia. His mother is very concerned and asks you what is wrong. What is the most likely diagnosis?

A) Myotonic dystrophy
B) Duchenne's muscular dystrophy
C) Down's syndrome
D) Myasthenia gravis
E) Becker's muscular dystrophy
A

B

Duchenne’s and Becker’s are X-linked recessive conditions, with Duchenne’s being most common, and Becker’s being less severe.

Myotonic dystrophy is autosomally dominant in inheritance, and presents with hypotonia from birth.

T21 would show dysmorphic features.

Myasthenia gravis can present as delayed walking, but would show fatigability.